You are on page 1of 71

TEST - 12

IA
S

1 The International Atomic Energy Agency (IAEA) reports to both the UN General
Assembly and the UN Security Council because
1. IAEA was established by the UN Charter.
2. It is under the direct control of the UN.
Which of the above is/are correct?
A. 1 only
B. 2 only
C. Both 1 and 2
D. None
User Answer :
Correct Answer : D
Answer Justification :

TS

Justification & Learning:Though established independently of the United Nations


through its own international treaty, the IAEA Statute, the IAEA reports to both the
United Nations General Assembly and Security Council. However, it is not under
the direct control of the UN. So, both 1 and 2 are wrong.
The IAEA has its headquarters in Vienna.

The IAEA serves as an intergovernmental forum for scientific and technical


cooperation in the peaceful use of nuclear technology and nuclear power worldwide.

Q Source: Page 85: Chapter 6: 12th NCERT: Contemporary World Politics

IN

SI

2 Consider the following about the International Court of Justice (ICJ).


1. It provides advisory opinions on legal questions submitted to it by the UN General
Assembly.
2. UN Security Council can enforce its rulings.
3. No two judges may be nationals of the same country in the ICJ.
4. Its judges are appointed for five-year terms by the UN Security Council.
Select the correct answer using the codes below.
A. 1, 2 and 3 only
B. 3 and 4 only
C. 1, 2 and 4 only
D. 1, 2, 3 and 4
User Answer :
Correct Answer : A
Answer Justification :
Justification: Statement 1: ICJ is the primary judicial branch of the United Nations

(C) Insights Active Learning. | All rights reserved.

www.insightsias.com

TEST - 12

(UN) in the Hague. It settles legal disputes submitted to it by states and provides
advisory opinions on legal questions submitted to it by duly authorized international
branches, agencies, and the UN General Assembly.

IA
S

Statement 2: Chapter XIV of the United Nations Charter authorizes the UN Security
Council to enforce Court rulings. However, such enforcement is subject to the veto
power of the five permanent members of the Council, which the United States used
in the Nicaragua case.
Statement 3 and 4: The ICJ is composed of fifteen judges elected to nine-year terms
by the UN General Assembly and the UN Security Council from a list of people
nominated by the national groups in the Permanent Court of Arbitration.

TS

According to Article 9, the membership of the Court is supposed to represent


the "main forms of civilization and of the principal legal systems of the
world". Essentially, that has meant common law, civil law and socialist law
(now post-communist law).
There is an informal understanding that the seats will be distributed by
geographic region
The five permanent members of the United Nations Security Council (France,
Russia, China, the United Kingdom, and the United States) always have a
judge on the Court (except for 1967-1985 China).

Q Source: Page 85: Chapter 6: 12th NCERT: Contemporary World Politics

IN

SI

3 Consider the following statements.


1. It is India's largest inland salt lake.
2. It is geographically a land locked river basin.
3. It has been designated as a Ramsar site.
The above refer to?
A. Bhimtal Lake
B. Bhitarkanika Lake
C. Sambhar Lake
D. Shakambari Lake
User Answer :
Correct Answer : C
Answer Justification :
Learning: Sambhar has been designated as a Ramsar site (recognized wetland of
international importance) because the wetland is a key wintering area for tens of
thousands of flamingos and other birds that migrate from northern Asia.

(C) Insights Active Learning. | All rights reserved.

www.insightsias.com

TEST - 12

The specialized algae and bacteria growing in the lake provide striking water
colours and support the lake ecology that, in turn, sustains the migrating waterfowl.
It is not part of the Ganga river basin area and is geographically a separate land
locked river basin.

IA
S

It is the source of most of Rajasthan's salt production.


Q Source: SET A: Improvisation: Q4: CSP 2009

TS

4 CNX Nifty is used extensively by investors in India and around the world as a barometer
of the Indian capital markets. It is prepared by
A. Reserve Bank of India (RBI)
B. Securities and Exchange Board of India (SEBI)
C. National Stock Exchange of India
D. Department of Financial Services, Union Government
User Answer :
Correct Answer : C
Answer Justification :

Learning: The NIFTY 50 covers 22 sectors of the Indian economy and offers
investment managers exposure to the Indian market in one portfolio.
The base period for the CNX Nifty index is 1995.

SI

NSE was the first exchange in the country to provide a modern, fully automated
screen-based electronic trading system which offered easy trading facility to the
investors spread across the length and breadth of the country.

IN

NSE was set up by a group of leading Indian financial institutions at the behest of
the government of India to bring transparency to the Indian capital market.
Q Source: SET A: Improvisation: Q7: CSP 2009

5 A High-level government committee headed by Deepak Parekh that submitted its report
in 2014 was set up to give recommendations on which sector?
A. Public Sector Bank (PSB) Reforms
B. Infrastructure financing
C. Railways modernization
D. Housing shortage

(C) Insights Active Learning. | All rights reserved.

www.insightsias.com

TEST - 12

User Answer :
Correct Answer : B
Answer Justification :

IA
S

Learning: The Deepak Parekh committee was tasked with estimating the
investments to be made by the public and private sectors in major core sectors and
suggest areas that are suitable for public and private investments.
It was also expected to suggest changes in the business environment and the capital
markets framework to facilitate higher flows of savings into infrastructure projects.
Some of its proposals are as follows:-

TS

Establish a PPP model for power distribution, starting first with the cities
Modernise public sector distribution companies. The viability gap for such
companies should be funded by the Central government
Tariffs should be rationalized with a grading system that distinguishes
between consumers depending on their paying capacity.
Privatise coal mining wherein Coal India or its arms/subsidiaries can retain
ownership of mines
http://articles.economictimes.indiatimes.com/2014-05-06/news/49661704_1_infrastr
ucture-projects-new-infrastructure-deepak-parekh

Q Source: SET A: Improvisation: Q11: CSP 2009

IN

SI

6 Consider the following matches of Buddhist Monasteries with their locations.


1. Tabo Monastery : Spiti Valley
2. Kardang Monastery : Nubra Valley
3. Rumtek Monastery : Lhasa
Select the correct answer using the codes below.
A. 1 and 2 only
B. 3 only
C. 2 only
D. 1 only
User Answer :
Correct Answer : D
Answer Justification :
Learning: It is from here that UPSC lifted the question on Buddhist monasteries
and locations in CSP 2014.

(C) Insights Active Learning. | All rights reserved.

www.insightsias.com

TEST - 12

Statement 1: It is located in the Tabo village of Spiti Valley, Himachal Pradesh. It


was founded in 996 CE in the Tibetan year of the Fire Ape.
Tabo is noted for being the oldest continuously operating Buddhist enclave in both
India and the Himalayas. A large number of frescoes displayed on its walls depict
tales from the Buddhist pantheon.

IA
S

Statement 2: Kardang Monastery or Gompa is a famous Drukpa Lineage monastery,


and is the most important monastery the Lahaul valley, India. The monastery is a
huge white building bedecked with prayer flags.

TS

Statement 3: The monastery is currently the largest in Sikkim. It is home to the


community of monks and where they perform the rituals and practices of the Karma
Kagyu lineage. A golden stupa contains the relics of the 16th Karmapa. Opposite
that building is a college, Karma Shri Nalanda Institute for Higher Buddhist Studies.
Q Source: SET A: Improvisation: Q15: CSP 2009

IN

SI

7 India has recently ratified the Convention on Supplementary Compensation for Nuclear
Damage (CSC), 1997. Consider the following about CSC.
1. It sets parameters on a nation's financial liability in case of a nuclear accident.
2. It fulfils the commitments India made in the 2005 Nuclear Agreement with the
USA.
Select the correct answer using the codes below.
A. 1 only
B. 2 only
C. Both 1 and 2
D. None
User Answer :
Correct Answer : B
Answer Justification :
Justification: Statement 1: It does not set parameters on a nation's financial liability
but that of the nuclear plant operator. It seeks to establish a uniform global legal
regime for compensation to victims in the unlikely event of a nuclear accident.
Statement 2: India had signed it in 2010 for delivering its commitments for
stemming the landmark 2005 nuclear agreement with the United States. It finally
ratified it recently.
This article explains it.

(C) Insights Active Learning. | All rights reserved.

www.insightsias.com

TEST - 12

http://www.ibtimes.com/india-us-nuclear-agreement-india-clarifies-says-supplier-no
t-directly-liable-case-1809012

IA
S

Q Source:
http://www.mea.gov.in/press-releases.htm?dtl/26324/India+submits+the+Instrument
+of+Ratification+of+the+Convention+on+Supplementary+Compensation+for+Nucl
ear+Damage+CSC+1997

TS

8 Which of the following best describes 'Gatka'?


A. A martial art associated with the Sikh history
B. A traditional theatre form of Kashmir where wit and parodies are used to
induce laughter
C. Folk theatre of Kerala performed by tribals
D. Narration of mythical plays with emphasis on hand gestures and eye
movements
User Answer :
Correct Answer : A
Answer Justification :

Learning: It an integral part of an array of Sikh Shastar Vidiya developed during


15th century.

It is generally at public display during religious processions.

SI

Gatka is a style of stick fighting between two or more practitioners, with wooden
sticks (called Soti) intended to simulate swords.
It is a unique art to defend & display fighting skills and exercise self-control.

IN

The Government of Punjab and School Games Federation of India (SGFI) have also
incorporated the Gatka game into the school games.
Q Source: SET A: Improvisation: Q18: CSP 2009

9 Bhimbetaka rock art and paintings belong to which of these periods?


1. Upper Paleolithic
2. Mesolithic
3. Chalcolithic
4. Medieval
Select the correct answer using the codes below.

(C) Insights Active Learning. | All rights reserved.

www.insightsias.com

TEST - 12

IA
S

A. 1 and 2 only
B. 2 only
C. 3 and 4 only
D. 1, 2, 3 and 4
User Answer :
Correct Answer : D
Answer Justification :
Justification: The drawings and paintings can be classified under seven different
periods.

SI

TS

Period I - (Upper Paleolithic): These are linear representations, in green and


dark red, of huge figures of animals such as bison, tigers and rhinoceroses.
Period II - (Mesolithic): Comparatively small in size the stylised figures in
this group show linear decorations on the body. In addition to animals there
are human figures and hunting scenes
Period III - (Chalcolithic) Similar to the paintings of the Chalcolithic, these
drawings reveal that during this period the cave dwellers of this area were in
contact with the agricultural communities of the Malwa plains, exchanging
goods with them.
Period IV & V - (Early historic): The figures of this group have a schematic
and decorative style and are painted mainly in red, white and yellow. The
association is of riders, depiction of religious symbols
Period VI & VII - (Medieval) : These paintings are geometric linear and more
schematic, but they show degeneration and crudeness in their artistic style.
The colors used by the cave dwellers were prepared by combining
manganese, hematite and wooden coal.

IN

Q Source: Page 7: Chapter 1: 11th NCERT: An Introduction to Indian Arts

10 Tropic of Capricorn passes through which of these countries?


1. Indonesia
2. Sudan
3. Chile
Select the correct answer using the codes below.
A. 1 and 2 only
B. 2 and 3 only
C. 3 only
D. 2 only
User Answer :

(C) Insights Active Learning. | All rights reserved.

www.insightsias.com

TEST - 12

Correct Answer : C
Answer Justification :
Learning: Countries through which tropic of Capricorn passes through are as
follows in west to east direction: Chile; Argentina; Paraguay; Brazil; Namibia;
Botswana; South Africa; Mozambique; Madagascar; Australia.

IA
S

Of the three major lines, Only Tropic of Cancer passes through India.

Tropic of cancer passing through Indian states: Gujarat; Rajasthan; Madhya


Pradesh; Chhattisgarh; Jharkhand; West Bengal; Tripura; Mizoram.
Q Source: SET A: Improvisation: Q34: CSP 2009

TS

11 Natural rate of population increase per thousand was highest in India during which of
the following decades?
A. Post-liberalization period
B. Post-Bangladesh war period
C. Decade immediately after India's independence
D. Post-Kargil war period
User Answer :
Correct Answer : B
Answer Justification :

SI

Justification: During 1971-1981 natural increase rate per thousand was around 22.
It was a period of rapid growth of population.
Post-liberalization it hovered around 17-20.

IN

Immediately after independence it was in the range of 13-19.


Post-Kargil war it was around 17. After 2001 population growth slowed down.
To sum it percentage terms, during 1921 to 1951 A.D., there was an increase in
population by 12% and from 1951 to 1981, the growth showed an increase by
24.75%.
Q Source: SET A: Improvisation: Q40: CSP 2009

12 Consider the following about White Dwarfs.


1. They are the lightest forms of matter.

(C) Insights Active Learning. | All rights reserved.

www.insightsias.com

TEST - 12

IA
S

2. They can give an idea about the age of the Universe.


3. White dwarfs are the end product of black holes.
Select the correct answer using the codes below.
A. 1 and 2 only
B. 3 only
C. 2 only
D. 1, 2 and 3
User Answer :
Correct Answer : C
Answer Justification :

Justification: White dwarfs are one of the densest forms of matter, surpassed only
by neutron stars and black holes. So, 1 is wrong.

TS

A low or medium mass star (with mass less than about 8 times the mass of our Sun)
will become a white dwarf. Stars that have a lot of mass may end their lives as black
holes or neutron stars. So, 3 is wrong. v

White dwarfs can tell us about the age of the Universe. If we can estimate the time it
takes for a white dwarf to cool into a black dwarf, that would give us a lower limit
on the age of the Universe and our galaxy. So, 2 is correct.

Q Source: SET A: Improvisation: Q44: CSP 2009

IN

SI

13 Consider the following about the Third Round Table Conference.


1. Indian National Congress (INC) did not attend it
2. White paper published as a result of the conference became the basis of the
Government of India Act 1935.
3. British proposed the idea of a 'commonwealth' of colonial nations in this conference.
Select the correct answer using the codes below.
A. 1 and 2 only
B. 3 only
C. 2 only
D. 1, 2 and 3
User Answer :
Correct Answer : A
Answer Justification :
Justification: Statement 1: The Labour Party from Britain and the Indian National
Congress refused to attend. So, 1 is correct.

(C) Insights Active Learning. | All rights reserved.

www.insightsias.com

TEST - 12

Statement 2: The recommendations of conference were published in a White Paper


in March 1933 and debated in Parliament afterwards. A Joint Select Committee was
formed to analyse the recommendations and formulate a new GoI Act 1935 for
India. So, 2 is correct.

IA
S

Statement 3: The Commonwealth dates back to the mid-20th century with the
decolonisation of the British Empire through increased self-governance of its
territories. It was formally constituted by the London Declaration in 1949. So, 3 is
wrong.
Q Source: SET A: Improvisation: Q44: CSP 2009

SI

TS

14 Which among the following is/are member(s) of the National Water Resources Council
(NWRC)?
1. Chief Ministers of all States
2. Union Minister of Water Resources
3. Chairman, NITI Aayog
Select the correct answer using the codes below.
A. 1 and 2 only
B. 3 only
C. 2 only
D. 1, 2 and 3
User Answer :
Correct Answer : A
Answer Justification :
Learning: National Water Resources Council was set up by the Government of
India in 1983.

IN

The Prime Minister is the Chairman, Union Minister of Water Resources is the
Vice-Chairman, and Minister of State for Water Resources, concerned Union
Ministers/ Ministers of State, Chief Ministers of all States & Lieutenant Governors/
Administrators of the Union Territories are the Members. Secretary, Ministry of
Water Resources is the Secretary of the Council.
The Government of India constituted a National Water Board in September, 1990
under the Chairmanship of Secretary, Ministry of Water Resources to review the
progress achieved in implementation of the National Water Policy and to report the
progress to the National Water Resources Council from time to time.
Q Source: SET A: Improvisation: Q50: CSP 2009

(C) Insights Active Learning. | All rights reserved.

www.insightsias.com

10

TEST - 12

IA
S

15 The origins of Anekantavada can be traced back to the teachings of Mahavira.


Anekantavada means
A. Human beings cross many lives in the universal flux.
B. No single point of view is the complete truth.
C. Principle of Karma binds all mortal beings.
D. Multiple ways towards salvation lead to nowhere.
User Answer :
Correct Answer : B
Answer Justification :

TS

Learning: Jains contrast all attempts to arrogantly proclaim the sole monopoly on
truth with andhagajany?yah, which can be illustrated through the parable of the
"blind men and an elephant". In this story, each blind man felt a different part of an
elephant (trunk, leg, ear, etc.).
All the men claimed to understand and explain the true appearance of the elephant,
but could only partly succeed, due to their limited perspectives.

This principle is more formally stated by observing that objects are infinite in their
qualities and modes of existence, so they cannot be completely grasped in all aspects
and manifestations by finite human perception (this is the Absolute Truth).

According to the Jains, only the Kevalisomniscient beingscan comprehend objects in


all aspects and manifestations; others are only capable of partial knowledge.
Consequently, no single, specific, human view can claim to represent absolute truth.

SI

Q Source: SET A: Improvisation: Q57: CSP 2009

IN

16 Aruna Asif Ali was associated with which of the following?


1. Quit India Movement
2. Salt Satyagraha
3. National Federation of Indian Women
Select the correct answer using the codes below.
A. 1 and 2 only
B. 1 and 3 only
C. 2 and 3 only
D. 1, 2 and 3
User Answer :
Correct Answer : D
Answer Justification :

(C) Insights Active Learning. | All rights reserved.

www.insightsias.com

11

TEST - 12

Justification: She is widely remembered for hoisting the Indian National Congress
flag at the Gowalia Tank maidan in Bombay during the Quit India Movement, 1942.
So, 1 is correct.
She became an active member of Congress Party after marrying Asaf Ali and
participated in public processions during the Salt Satyagraha. So, 2 is correct.

IA
S

In 1954, she helped form the National Federation of Indian Women, the women's
wing of CPI but left the party in 1956 following Nikita Khrushchev's disowning of
Stalin. So, 3 is correct.
Q Source: SET A: Improvisation: Q63: CSP 2009

TS

17 Camphor tree is most likely to be naturally found in


A. Evergreen forests
B. Boreal forests
C. Temperate deciduous forests
D. Mediterranean vegetation
User Answer :
Correct Answer : A
Answer Justification :

Learning: Cinnamomum camphora (from where Camphor can be obtained) is a


massive broad leaved evergreen tree.

SI

Native to the wet forests of subtropical and tropical Asia, the camphor tree,
Cinnamomum camphora, has been planted across the world's warm temperate,
subtropical, and tropical regions.

IN

Its defining feature is the essential oil that can be extracted from its tissues and the
mixture of aromatic chemicals that give the oil its pungent odor. Of these, camphor
is the major ingredient- a highly aromatic terpene like chemical.
Q Source: SET A: Improvisation: Q73: CSP 2009

18 Consider the following about Shompen tribe.


1. They are the indigenous people of Lakshadweep Islands.
2. They have been classified as a Primitive Tribal Group (PTG).
3. They speak South Indian languages.
4. They are the descendants of Jarawa tribal lineage.

(C) Insights Active Learning. | All rights reserved.

www.insightsias.com

12

TEST - 12

IA
S

Select the correct answer using the codes below.


A. 2 only
B. 1 and 3 only
C. 2 and 3 only
D. 1, 2 and 4 only
User Answer :
Correct Answer : A
Answer Justification :

Justification:Statement 1: They are the indigenous people of the interior of Great


Nicobar Island.

TS

Statement 2 and 4: The Shompen are a designated Scheduled Tribe and a PTG. Not
as well known as the Jarawa tribe of the Andaman islands, the Shompen are as
precariously poised on the brink of extinction as the four other hunter-gatherer tribes
(the Jarawa, the Andamanese, the Onge and the Sentinelese

Statement 3: The Shompen languages, of which there are at least two, are very little
known, but appear to be unrelated to Nicobarese, an isolated group of Austroasiatic
languages, and perhaps even to each other. They may constitute a language isolate.
So, 3 is wrong.

http://www.thehindu.com/todays-paper/tp-features/tp-sundaymagazine/tragedy-of-th
e-shompen/article2275375.ece

SI

Q Source: SET A: Improvisation: Q81: CSP 2009

IN

19 Consider the following with reference to the United Nations Economic and Social
Council (UNECOSOC).
1. Members are appointed on rotation basis by the United Nations Secretary General.
2. Seats on the Council are based on geographical representation.
3. International Labour Organization (ILO) is one of its sub-agencies.
Select the correct answer using the codes below.
A. 2 only
B. 1 and 3 only
C. 2 and 3 only
D. 1, 2 and 3
User Answer :
Correct Answer : A
Answer Justification :

(C) Insights Active Learning. | All rights reserved.

www.insightsias.com

13

TEST - 12

Justification: ILO is an autonomous organization working with the United Nations


coordinating with other specialized agencies of the UN and UNECOSOC. So, 3 is
wrong.

IA
S

The Council has 54 member states out of the 193 UN member states, which are
elected by the United Nations General Assembly for overlapping three-year terms.
Seats on the Council are based on geographical representation with 18 allocated to
African states, 13 to Asian states, 8 to East European states, 13 to Latin American
and Caribbean states and 13 to West European and other states.
Q Source: SET A: Improvisation: Q80: CSP 2009

SI

TS

20 Consider the following with reference to the Cartagena Protocol on Biosafety.


1. It seeks to promote genetically modified organisms (GMOs) in order to ensure
biological diversity.
2. It restricts countries from banning any imports of GMOs.
3. It acts as a supplement to the Convention on Biological Diversity (CBD).
Select the correct answer using the codes below.
A. 1 and 2 only
B. 3 only
C. 2 and 3 only
D. 1, 2 and 3
User Answer :
Correct Answer : B
Answer Justification :

IN

Justification and Learning: The Cartagena Protocol on Biosafety to the


Convention on Biological Diversity is an international agreement on biosafety as a
supplement to the Convention on Biological Diversity effective since 2003. So, 3 is
correct.
The Biosafety Protocol seeks to protect biological diversity from the potential
risks posed by genetically modified organisms resulting from modern
biotechnology. So, 1 is incorrect.
The Biosafety Protocol makes clear that products from new technologies
must be based on the precautionary principle and allow developing nations to
balance public health against economic benefits.
It will for example let countries ban imports of a genetically modified
organisms if they feel there is not enough scientific evidence that the product
is safe and requires exporters to label shipments containing genetically

(C) Insights Active Learning. | All rights reserved.

www.insightsias.com

14

TEST - 12

altered commodities such as corn or cotton. So, 2 is incorrect.


It is implemented by the Ministry of Environment and Forests.
Q Source: SET A: Improvisation: Q80: CSP 2009

TS

IA
S

21 Consider the following about Sangeet Natak Akademi Award fellowship.


1. It is the highest honour in the performing arts conferred by the Government of India.
2. Allauddin Khan was one of the first awardees.
3. The Awardees are entitled to a financial grant for establishing a private autonomous
performing arts institution.
Select the correct answer using the codes below.
A. 1 and 2 only
B. 3 only
C. 2 and 3 only
D. 1, 2 and 3
User Answer :
Correct Answer : A
Answer Justification :

Justification & Learning:The Akademi Ratna (Fellow) may be conferred, as per


the existing constitutional provision under Rule 12 (vi) of the Rules and Regulations
of the Akademi, which reads as under:

IN

SI

"To elect, by a majority of at least three-fourth of the members present and voting,
artists of outstanding merit in the field of Music, Dance and Drama or such persons
as have rendered outstanding service to the cause of music, dance & drama through
their scholarship, research or original contributions as Fellows of the Akademi,
provided they have been recommended for the elction by the Executive Board and
provided further the number of Fellows shall at no time exceed thirty."
It is subject to the approval of the Department of Culture, Government of
India.
The fellowship was established in 1954 and the first elected fellows were the
Carnatic musicians Karaikudi Sambasiva Aiyer and Ariyakudi Ramanuja
Iyengar, the Hindustani musicians Allauddin Khan and Hafiz Ali Khan, and
movie and theatre actor Prithviraj Kapoor.

Q Source: SET A: Improvisation: Q93: CSP 2009 + Recent issue of Returning of


Awards of the Akademi

(C) Insights Active Learning. | All rights reserved.

www.insightsias.com

15

TEST - 12

TS

IA
S

22 Consider the following statements about the administrative setup of the Government of
India.
1. The Ministries/Departments of the Government are created by the President on the
advice of the Prime Minister.
2. Each of the Ministries is assigned to a Minister by the President on the advice of the
Prime Minister.
3. The rules for the allocation of business of the Government of India are made by the
President.
Select the correct answer using the codes below.
A. 1 and 2 only
B. 3 only
C. 2 and 3 only
D. 1, 2 and 3
User Answer :
Correct Answer : D
Answer Justification :

Justification & Learning: The Government of India (Allocation of Business)


Rules, 1961 are made by the President of India under Article 77 of the Constitution
for the allocation of business of the Government of India. So, 3 is correct.

The Ministries/Departments of the Government are created by the President on the


advice of the Prime Minister under these Rules. The business of the Government are
transacted in the Ministries/Departments, Secretariats and offices (referred to as
'Department') as per the distribution of subjects specified in these Rules.

SI

Each of the Ministries is assigned to a Minister by the President on the advice of the
Prime Minister. Each department is generally under the charge of a Secretary to
assist the Minister on policy matters and general administration.

IN

Q Source: SET A: Improvisation: Q94: CSP 2009

23 Laser diodes are used in which of these device(s)?


1. Baking Oven
2. Barcode Readers
3. Air conditioner
4. Blu-ray Discs
Select the correct answer using the codes below.
A. 1 and 4 only
B. 1, 2 and 3 only

(C) Insights Active Learning. | All rights reserved.

www.insightsias.com

16

TEST - 12

C. 2 and 4 only
D. 1, 2, 3 and 4
User Answer :
Correct Answer : C
Answer Justification :

IA
S

Justification: Both DVDs and Blu-ray Discs use laser diode, but in DVD it is red
laser, in Blu-ray Discs it is blue (violet) laser. The laser is focused to small areas,
thus enabling it to read information recorded in pits of the discs. So, 4 is correct.
Many Barcode readers use a laser beam as the light source and typically have either
a reciprocating (return) mirror or a rotating prism to scan the laser beam back and
forth across the bar code.

TS

A photo-diode is used to measure the intensity of the light reflected back from the
bar code to read the pattern. So, 2 is correct.
Air Conditioners use compressors and fluid circulation to cool a place. A baking
oven doesn't use laser diodes.

Q Source: CSP 2008

IN

SI

24 How is the UN Secretary-General selected?


A. Elected on a majority basis by the UN General Assembly (UNGA)
B. Appointed by the UN Security Council (UNSC) based on the discretion of the
P-5
C. Appointed by the General Assembly on the recommendations of the UNSC
D. Nominated by a body of former UN Secretary Generals
User Answer :
Correct Answer : C
Answer Justification :
Learning: The Secretary-General of the United Nations (UNSG) is the head of the
United Nations Secretariat, one of the principal organs of the United Nations.
The Secretary-General also acts as the de facto spokesperson and leader of the
United Nations.
Responsibilities of the Secretary-General are further outlined in Articles 98 through
100, which states that they shall act as the officer "in all meetings of the General
Assembly, of the Security Council, of the Economic and Social Council and the

(C) Insights Active Learning. | All rights reserved.

www.insightsias.com

17

TEST - 12

Trusteeship Council, and shall perform other functions as are entrusted to him by
these organs".
They are responsible, according to Article 99, for making an annual report to the
General Assembly as well as notifying the Security Council on matters which "in
their opinion may threaten the maintenance of international peace and security".

TS

25 The Shramana tradition is related to


A. The path of seeking the ultimate truth
B. Social service amongst the most vulnerable
C. Occult practices in the Tibetan region
D. Martial arts taught to princes in Gurukuls
User Answer :
Correct Answer : A
Answer Justification :

IA
S

Q Source: Page 85: Chapter 6: 12th NCERT: Contemporary World Politics

Learning: Shramana means "seeker, one who performs acts of austerity, ascetic".

The term refers to several Indian religious movements parallel to but separate from
the historical Vedic religion.
The ?rama?a tradition includes Jainism of 9th-century BCE, Buddhism of 6thcentury BCE, and others such as Ajivika and Carvaka.

IN

SI

The Srama?a movements arose in the same circles of mendicants in ancient India
that led to the development of Yogic practices, as well as the popular concepts in all
major Indian religions such as sa?s?ra (the cycle of birth and death) and moksha
(liberation from that cycle).
Q Source: Page 128-30: Chapter 8: 11th NCERT: An Introduction to Indian Arts

26 Consider the following about the Animal Welfare Board of India.


1. It is an advisory body established by an executive order.
2. It is chaired by the Prime Minister of India.
3. The Board provides financial assistance to recognised Animal Welfare
Organisations(AWOs).
Select the correct answer using the codes below.
A. 1 and 2 only

(C) Insights Active Learning. | All rights reserved.

www.insightsias.com

18

TEST - 12

IA
S

B. 2 and 3 only
C. 3 only
D. None of the above
User Answer :
Correct Answer : C
Answer Justification :
Justification:Statement 1: The Animal Welfare Board of India is a statutory
advisory body on Animal Welfare Laws and promotes animal welfare in the
country. It was established in 1962 under the Prevention of Cruelty to Animals Act,
1960. So, 1 is incorrect.
Statement 2: National Board for Wildlife is chaired by the PM, not the welfare
board.

TS

Statement 3: It works to ensure that animal welfare laws in the country are followed
and provides grants to Animal Welfare Organisations.

The Board oversees Animal Welfare Organisations (AWOs) by granting recognition


to them if they meet its guidelines.

Q Source:SET A: Improvisation: Q91: CSP 2009

IN

SI

27 Consider the following about Ashta Pradhan.


1. It was a council of eight ministers that administered the Maratha Empire.
2. It did not involve itself in military campaigns.
3. No appointment to the body was made on a hereditary basis.
4. Its role did not concern with dispensing justice on civil and criminal matters.
Select the correct answer using the codes below.
A. 1, 2 and 4 only
B. 1 only
C. 2, 3 and 4 only
D. 3 only
User Answer :
Correct Answer : B
Answer Justification :
Justification & Learning: The Ashta Pradhan was designed to encompass all the
primary administrative functions of the state, with each minister being given charge
of one role in the administration. Ministerial designations were drawn from the
Sanskrit language; the eight ministerial roles were as follows:

(C) Insights Active Learning. | All rights reserved.

www.insightsias.com

19

TEST - 12

Q Source:SET A: Q116: CAPF 2011

TS

IA
S

28 Consider the following about the World Economic Forum (WEP).


1. It is an intergovernmental organization committed to improving economic policy
making.
2. All its meetings are conducted in Geneva.
3. It publishes the annual report "Global Economic Prospects".
4. The Global Health Initiative was launched at one of its meetings.
Select the correct answer using the codes below.
A. 1 only
B. 4 only
C. 1, 2 and 3 only
D. None of the above
User Answer :
Correct Answer : B
Answer Justification :

Justification:Statement 1: It is a Swiss non-profit foundation, based in Cologny,


Geneva. Recognized by the Swiss authorities as the international institution for
public-private cooperation, its mission is cited as "committed to improving the state
of the world by engaging business, political, academic, and other leaders of society
to shape global, regional, and industry agendas".

SI

Statement 2: The Forum is best known for its annual winter meeting in Davos, a
mountain resort in Graubnden, in the eastern Alps region of Switzerland. The
organization also convenes some six to eight regional meetings each year in
locations such as Latin America and East Asia, as well as undertaking two further
annual meetings in China and the United Arab Emirates.

IN

Statement 3: It is published by the World Bank.


Statement 4: The Global Health Initiative was launched by Kofi Annan at the annual
meeting in 2002. The GHI's mission was to engage businesses in public-private
partnerships to tackle HIV/AIDS, tuberculosis, malaria, and health systems. So, 4 is
correct.
Q Source:SET A: Improvisation: Q113: CSP 2009

29 Consider the following events in Indian economic history.


1. Life insurance in India was completely nationalized even before major commercial

(C) Insights Active Learning. | All rights reserved.

www.insightsias.com

20

TEST - 12

IA
S

banks were nationalized.


2. Banking regulation Act was passed before the establishment of the Reserve Bank of
India (RBI).
Select the correct answer using the codes below.
A. 1 only
B. 2 only
C. Both 1 and 2
D. None
User Answer :
Correct Answer : A
Answer Justification :

TS

Justification: Statement 1: The Insurance Act of 1938 was the first legislation
governing all forms of insurance to provide strict state control over insurance
business. Life insurance in India was completely nationalized in 1956, through the
Life Insurance Corporation Act. All 245 insurance companies operating then in the
country were merged into one entity, the Life Insurance Corporation of India.

Statement 2: The Banking Regulation Act, 1949 regulates all banking firms in India.
RBI commenced its operations from 1935 during the British Rule in accordance
with the provisions of the Reserve Bank of India Act, 1934.

Q Source: SET A: Q119: CSP 2009

IN

SI

30 Which of the following may have caused Dugong dugon numbers to decline in oceans?
1. Loss and degradation of Seagrass meadows
2. Fishing-related fatalities
3. Coastal pollution
Select the correct answer using the codes below.
A. 1 and 2 only
B. 2 and 3 only
C. 1 and 3 only
D. 1, 2 and 3
User Answer :
Correct Answer : d
Answer Justification :
Justification:Statement 1: The dugong is largely dependent on seagrass
communities for subsistence and is thus restricted to the coastal habitats which
support seagrass meadow.

(C) Insights Active Learning. | All rights reserved.

www.insightsias.com

21

TEST - 12

Sewage, detergents, heavy metal, hypersaline water, herbicides, and other waste
products all negatively affect seagrass meadows. Human activity such as mining,
trawling, dredging, land-reclamation, and boat propeller scarring also cause an
increase in sedimentation which smothers seagrass and prevents light from reaching
it.

IA
S

Statement 2: Entanglement in fishing nets has caused many deaths. As dugongs


cannot stay underwater for a very long period, they are highly prone to deaths due to
entanglement.
Statement 3: The fact that they live in shallow waters puts them under great pressure
from human activity, especially coastal pollution.

TS

Other reasons: The dugong has been hunted for thousands of years for its meat and
oil. Traditional hunting still has great cultural significance in several countries in its
modern range, particularly northern Australia and the Pacific Islands.
Q Source:SET A: Q132: CSP 2009

IN

SI

31 Which of these can be found at Ajanta Caves?


1. Boddhisatva Images
2. Mahaparinibbana image of Buddha
3. Tales from Avadanas
Select the correct answer using the codes below.
A. 1 and 2 only
B. 2 and 3 only
C. 1 and 3 only
D. 1, 2 and 3
User Answer :
Correct Answer : D
Answer Justification :

Justification: The themes of the paintings are the events from the life of the
Buddha, the Jatakas and the Avadanas. Some paintings such as Simhala Avadana,
Mahajanaka Jataka and Vidhurpundita Jataka cover the entire wall of the cave.
The other important paintings are the famous Padmapani and Vajrapan.
Figures in these caves are painted with considerable naturalism and there is no overstylisation.

(C) Insights Active Learning. | All rights reserved.

www.insightsias.com

22

TEST - 12

Q Source: Page 38-40: Chapter 4: 11th NCERT: An Introduction to Indian Arts

IA
S

32 The Anushilan Samiti formed in 1906 propounded


A. Constitutional methods to achieve administrative reforms
B. British support for bringing social reforms in India
C. Revolutionary violence as means for ending British rule in India
D. The doctrine of 'Council entry' to weaken the British raj from within
User Answer :
Correct Answer : C
Answer Justification :

Learning:It was a Bengali Indian organisation and arose from conglomerations of


local youth groups and gyms in Bengal.

TS

It had two prominent if somewhat independent arms in East and West Bengal
identified as Dhaka Anushilan Samiti and the Jugantar group respectively.

Between its foundations to its gradual dissolution through 1930s, the Samiti
collaborated with other revolutionary organisations in India and abroad.

SI

Led by notable revolutionaries of the likes of Aurobindo Ghosh, Rash Behari Bose
and Jatindranath Mukherjee, the Samiti was involved in a number of noted
incidences of revolutionary terrorism against British interests and administration in
India. These included the early attempts to assassinate Raj officials, the 1912
attempt on the life of Viceroy of India, as well as the Sedetious conspiracy during
World War I.
Q Source:SET A: Q115: CAPF 2011

IN

33 The Organisation for the Prohibition of Chemical Weapons (OPCW) is


A. An intergovernmental organisation that verifies the adherence to the
Chemical Weapons Convention
B. An international non-profit body that monitors global proliferation of
chemical weapons
C. A subsidiary organization of the UN reporting to the UNSC on matters
related to Chemical weapons
D. An arm of the Nuclear Suppliers Group (NSG) to prevent access of harmful
chemical weapons in wrong hands
User Answer :
Correct Answer : A

(C) Insights Active Learning. | All rights reserved.

www.insightsias.com

23

TEST - 12

Answer Justification :
Learning:The organisation was awarded the 2013 Nobel Peace Prize.
The activities of the OPCW and its core organisational structure are described in the
Chemical Weapons Convention (whose members are all in OPCW).

IA
S

The principal body is the conference of states parties, which normally is convened
yearly, and in which all countries participate and have equal voting rights.
All States Parties make contributions to the OPCW budget, based on a modified UN
scale of assessments.

TS

Q Source: Page 85: Chapter 6: 12th NCERT: Contemporary World Politics

SI

34 UNESCO's Memory of the World Register lists documentary heritage of world


significance. It includes which of these from India?
1. Rigveda
2. Mundaka Upanishad
3. Saiva Manuscript
Select the correct answer using the codes below.
A. 1 only
B. 2 and 3 only
C. 1 and 3 only
D. 1, 2 and 3
User Answer :
Correct Answer : C
Answer Justification :

IN

Justification:The Institute of Asian Studies Tamil Medical Manuscript Collection


was the first to be included.
Then Saiva Manuscript in Puducherry in 2005, and then Rigveda in 2007 was
included.
Saiva siddhanta, considered normative tantric Shaivism, provides the normative
rites, cosmology and theological categories of tantric Shaivism.
To date, 238 documentary heritages have been included in the Register, among them
recordings of folk music, ancient languages and phonetics, aged remnants of
religious and secular manuscripts, collective lifetime works of renowned giants of

(C) Insights Active Learning. | All rights reserved.

www.insightsias.com

24

TEST - 12

literature, science and music, copies of landmark motion pictures and short films,
and accounts documenting changes in the world's political, economic and social
stage.
Q Source:CSP 2008

TS

IA
S

35 Methane is emitted by
1. Wetlands
2. Raising of livestock
3. Landfills
4. Coal Mining
5. Oceans
Select the correct answer using the codes below.
A. 1, 2 and 5 only
B. 2, 3 and 4 only
C. 1 and 4 only
D. 1, 2, 3, 4 and 5
User Answer :
Correct Answer : D
Answer Justification :

Justification: Statement 1 and 5: Wetlands are the largest source, emitting CH4
from bacteria that decompose organic materials in the absence of oxygen. Smaller
sources include termites, oceans, sediments, volcanoes, and wildfires.

SI

Statement 2: Domestic livestock such as cattle, buffalo, sheep, goats, and camels
produce large amounts of CH4 as part of their normal digestive process.

IN

Statement 3: Methane is generated in landfills as waste decomposes and in the


treatment of wastewater.
Statement 4: Methane (CH4) is a gas formed as part of the process of coal
formation. When coal is mined, methane is released from the coal seam and the
surrounding disturbed rock strata. Methane can also be released as a result of natural
erosion or faulting.
Q Source:CSP 2008

36 Consider the following hills and their locations.


1. Shevroy Hills: Eastern Ghats

(C) Insights Active Learning. | All rights reserved.

www.insightsias.com

25

2. Cardamom Hills: Western Ghats


3. Anaimalai Hills: Garhjat Range
Select the correct answer using the codes below.
A. 1 and 2 only
B. 2 only
C. 1 and 3 only
D. 1, 2 and 3
User Answer :
Correct Answer : A
Answer Justification :

IA
S

TEST - 12

TS

Justification:Statement 1: The Servarayans form part of the southern ranges of the


Eastern Ghats System. it also represents the highest peak in southern part of the
Eastern Ghats, with the Solaikaradu peak.

Statement 2: They conjoin the Anaimalai Hills to the northwest, the Palni Hills to
the northeast and the Agasthyamalai Hills to the south as far as the Aryankavu. The
crest of the hills forms the boundary between Kerala and Tamil Nadu. Anamudi in
Eravikulam National Park, is the highest peak in Western Ghats and also the highest
point in India south of the Himalayas

Statement 3: They form a southern portion of the Western Ghats. Anamala /


Anaimalai Hills are south of where the Western Ghats are broken by the Palakkad
Gap, which in turn is south of the Nilgiri Hills. They border the state of Kerala on
the Southwest and the Cardamom Hills to the southeast.

SI

Q Source: CSP 2008

IN

37 Consider the following statements.


1. Human blood is normally acidic.
2. Human saliva is highly basic in nature.
Which of the above is/are correct?
A. 1 only
B. 2 only
C. Both 1 and 2
D. None
User Answer :
Correct Answer : D
Answer Justification :
Justification:Statement 1: Human blood stays in a very narrow pH range around

(C) Insights Active Learning. | All rights reserved.

www.insightsias.com

26

TEST - 12

(7.35 - 7.45 ). Below or above this range means symptoms and disease. If blood pH
moves to much below 6.8 or above 7.8, cells stop functioning and the patient dies.
The ideal pH for blood is 7.4

IA
S

Statement 2: The pH of saliva is usually between 6.5 - 7.5. After we chew and
swallow food it then enters the upper portion of the stomach which has a pH
between 4.0 - 6.5.
Q Source: CSP 2008

TS

38 The Yom Kippur War began when


A. An Arab coalition launched a joint surprise attack on the Israeli-occupied
territories
B. Assad's program of reform was severely protested by the Arab states
C. The Kurds rebelled against the central authority of Iraq
D. A general uprising rose against French occupations in Syria and Lebanon
User Answer :
Correct Answer : A
Answer Justification :

Justification: What is it? Probiotics are live bacteria and yeasts that are good for
your health, especially your digestive system. When you lose "good" bacteria in
your body (like after you take antibiotics, for example), probiotics can help replace
them.

IN

SI

Statement 1: Probiotics help move food through your gut. Commonly claimed
benefits of probiotics include the decrease of potentially pathogenic gastrointestinal
microorganisms, the reduction of gastrointestinal discomfort, the improvement of
the skin's function and the improvement of bowel regularity.
Statement 2: The consumption of probiotics may actually effect a modest benefit in
helping to control high blood pressure.
Q Source: CSP 2008

39 Probiotics are beneficial bacteria that primarily line your gut. They are also responsible
for
1. Nutrient absorption
2. Causing high blood pressure
Select the correct answer using the codes below.

(C) Insights Active Learning. | All rights reserved.

www.insightsias.com

27

TEST - 12

IA
S

A. 1 only
B. 2 only
C. Both 1 and 2
D. None
User Answer :
Correct Answer : A
Answer Justification :
Justification: What is it? Probiotics are live bacteria and yeasts that are good for
your health, especially your digestive system. When you lose "good" bacteria in
your body (like after you take antibiotics, for example), probiotics can help replace
them.

TS

Statement 1: Probiotics help move food through your gut. Commonly claimed
benefits of probiotics include the decrease of potentially pathogenic gastrointestinal
microorganisms, the reduction of gastrointestinal discomfort, the improvement of
the skin's function and the improvement of bowel regularity.

Statement 2: The consumption of probiotics may actually effect a modest benefit in


helping to control high blood pressure.

Q Source: CSP 2008

IN

SI

40 Krills are central to the survival of the marine life in Antarctica. Why?
A. They prevent overconcentration of iron in the ocean beds
B. They connect the marine food chain from phytoplanktons to larger marine
animals and fishes
C. They are a source of nutrition for the corals and seagrass
D. They filter the ocean water keeping it healthy for other marine animals
User Answer :
Correct Answer : B
Answer Justification :
Justification:They do not process iron in the ocean. Moreover, some concentration
of iron in the ocean is good for phytoplankton growth on which the Krills feed. So,
(a) would be wrong.
Krill are considered an important trophic level connection - near the bottom of the
food chain - because they feed on phytoplankton and to a lesser extent zooplankton,
converting these into a form suitable for many larger animals for whom krill makes
up the largest part of their diet. So, (b) is correct.

(C) Insights Active Learning. | All rights reserved.

www.insightsias.com

28

TEST - 12

They are not consumed by seagrass and corals. So, (c) is wrong.

Fig: Krill

TS

IA
S

Krills filter out their food from the water, not the water itself. So, (d) is also wrong.

SI

Q Source: Page 121: Chapter 8: 12th NCERT: Contemporary World Politics

IN

41 Babasaheb Ambedkar Special Collective Incentive Scheme has been launched by the
Maharashtra Government for
A. Economically backward Dalit entrepreneurs
B. Below Poverty Line (BPL) Dalits living in slums
C. Promoting rational inquiry in the institution of caste
D. Widow Dalits who are a part of Self-Help Groups (SHGs)
User Answer :
Correct Answer : A
Answer Justification :
Learning:
Under it, special facilities will be given to Dalit entrepreneurs besides the
present Collective Incentive Scheme 2013.

(C) Insights Active Learning. | All rights reserved.

www.insightsias.com

29

TEST - 12

IA
S

20% plots for small and medium scale industries will be reserved for the
Dalits within Maharashtra Industrial Development Corporation (MIDC).
Businessmen in Service and Commodities sector would be able to avail the
benefits of this scheme. It will be limited for only those industries where
people from Scheduled Castes (SCs) and Scheduled Tribes (STs) have 100%
investment.
Only one person from each family will be entitled to benefits under the
scheme and there will be no cap on income to launch any business.
Q Source:http://www.newsbharati.com/Encyc/2016/2/3/Maharashtra-Dalits

TS

42 Consider the following statements in respect of a jet engine and a rocket.


1. Both a jet engine and a rocket engine function by expelling hot gases opposite to the
direction of desired acceleration.
2. A rocket carries its own supply of oxygen for combustion.
3. A jet engine requires oxygen from the atmosphere for combustion.
Which of the statements given above is/are correct?
A. 1 and 2 only
B. 2 and 3 only
C. 1 and 3 only
D. 1, 2 and 3
User Answer :
Correct Answer : D
Answer Justification :

IN

SI

Learning: A rocket carries both fuel (which may be solid or liquid) and oxygen in
contrast to a jet engine. Therefore it does not suck in air from the front. It burns the
fuel with the oxygen, and eject it at very high velocities backward. This momentum
is used to both lift and propel the rocket. There are no wings for uplift. Any wings
are for steering purposes.
A rocket is generally much more powerful and wasteful than a jet engine. Most
airplanes cannot possibly climb vertically, while rockets are designed to do that. A
jet airplane cannot function outside the atmosphere, but a rocket can since it carries
its own fuel and oxygen.
Q Source:CSP 2008

43 Mahamastakabhisheka, a festival held once every twelve years, is related to


A. Jainism

(C) Insights Active Learning. | All rights reserved.

www.insightsias.com

30

TEST - 12

IA
S

B. Buddhism
C. Alvars
D. Nayanars
User Answer :
Correct Answer : A
Answer Justification :
Learning: It is held in the town of Shravanabelagola in Karnataka. The festival is
held in veneration of 57 ft high statue of the siddha Bahubali.
He was the son of Rishabhanatha, the first tirthankara of Jainism

The anointing last took place in 2006, and the next ceremony is going to take place
in 2018

TS

Jain rituals play a prominent part in Jainism. Worship, like in Hinduism, plays an
important role in Jainism too.
Adhara Abhisheka, a temple purification ceremony, is another important event.

Q Source: SET A: Improvisation: Q17: CSP 2009

IN

SI

44 The Nobel Prize in Physics 2015 was awarded for


A. Showing that electromagnetic waves can be transmitted in plasma like
temperatures.
B. Explaining micro-scale level phenomenon using Quantum Physics.
C. The discovery of neutrino oscillations, which shows that neutrinos have mass.
D. Finding out a distribution formula that measures the extent of both short and
long wavelengths from an underground source.
User Answer :
Correct Answer : C
Answer Justification :
Learning: The Nobel Prize in Physics 2015 was awarded jointly to Takaaki Kajita
and Arthur B. McDonald.
The sun was theorised to be powered by nuclear reactions in its core and these
produced neutrino particles.
Measurements of solar neutrino types were not consistent with models of the Sun's
interior that showed more neutrino production.

(C) Insights Active Learning. | All rights reserved.

www.insightsias.com

31

TEST - 12

They demonstrated that the neutrinos from the Sun were not disappearing on their
way to Earth and can change their mass becoming undetected. This resolved the sun
neutrino problem.
Q Source: CSP 2007 and Current Affairs

TS

IA
S

45 Who among the following is/are the members of the Shanghai Cooperation
Organization (SCO)?
1. Japan
2. Saudi Arabia
3. Afghanistan
4. South Korea
Select the correct answer using the codes below.
A. 1 and 4 only
B. 2 and 3 only
C. 3 only
D. None of the above
User Answer :
Correct Answer : D
Answer Justification :

Justification: It is a Eurasian political, economic and military organisation which


was founded in 2001 in Shanghai by the leaders of China, Kazakhstan, Kyrgyzstan,
Russia, Tajikistan, and Uzbekistan.

SI

These countries, except for Uzbekistan had been members of the Shanghai Five,
founded in 1996; after the inclusion of Uzbekistan in 2001, the members renamed
the organisation.

IN

In 2015, the SCO decided to admit India and Pakistan as full members, and they are
expected to join by 2016.
Q Source:CSP 2007

46 Consider the following about Participatory Notes (PN).


1. It is investment by Foreign Institutional Investors (FIIs) that have relaxed KYC
norms.
2. Only individuals investing from tax havens are allowed to invest through PN in
India.
Which of the above is/are correct?

(C) Insights Active Learning. | All rights reserved.

www.insightsias.com

32

TEST - 12

IA
S

A. 1 only
B. 2 only
C. Both 1 and 2
D. None
User Answer :
Correct Answer : A
Answer Justification :
Justification: Statement 1: For a person to invest even in one share, several KYC
(know your customer) forms have to be filled up, and PAN numbers and proof of
address, etc., provided. For the PN investor the system is totally silent on even
elementary information. The FIIs issue PNs to funds/companies whose identity is
not known to the Indian authorities.

TS

Statement 2: It can be invested from anywhere.


Read these two links to understand more about PN and recent trends.

http://www.thehindubusinessline.com/todays-paper/why-participatory-notes-are-dan
gerous/article1672845.ece
https://en.wikipedia.org/wiki/Participatory_note#Trends

Q Source: CSP 2007

IN

SI

47 Consider the following about the International Date Line.


1. It is a straight line that connects North Pole and South Pole.
2. It demarcates the change of one calendar day to the next.
3. The line passes through the Indian Ocean.
Select the correct answer using the codes below.
A. 2 only
B. 1 and 2 only
C. 2 and 3 only
D. 1, 2 and 3
User Answer :
Correct Answer : A
Answer Justification :
Justification:Statement 1: It is not a straight line. For parts of its length, the IDL
follows the meridian of 180' longitude, roughly down the middle of the Pacific
Ocean. To avoid crossing nations internally, the IDL deviates west around the US

(C) Insights Active Learning. | All rights reserved.

www.insightsias.com

33

TEST - 12

Aleutian Islands, separating them from islands in the far east of Russia, and further
south, it deviates east around various island nations in the Pacific such as Kiribati,
Samoa, Tonga and Tokelau.

IA
S

Statement 2: A person who goes around the world from east to west (the same
direction as Magellan's voyage) would gain or set their clock back one hour for
every 15' of longitude crossed, and would gain 24 hours for one circuit of the globe
from east to west if they did not compensate by setting their clock forward one day
when they crossed the IDL.
In contrast, a west-to-east circumnavigation of the globe loses an hour for every 15'
of longitude crossed but gains back a day when crossing the IDL. The IDL must
therefore be observed in conjunction with the Earth's time zones: on crossing it in
either direction, the calendar date is adjusted by one day.

Q Source: CSP 2007

TS

Statement 3: It passes through the middle of the pacific ocean.

IN

SI

48 The Wavell Plan arrived at the Simla Conference 1945 provided for which of the
following?
1. Indianization of the Viceroy's Executive Council
2. Removing any caste and religion based quota in the Executive Council
3. Partition of India
Select the correct answer using the codes below.
A. 1 and 2 only
B. 2 and 3 only
C. 1 only
D. 1, 2 and 3
User Answer :
Correct Answer : C
Answer Justification :
Justification: Statement 1: As per the Plan, all the members of the Council, except
the Viceroy and the Commander-in-Chief, would be Indians.
Statement 2: It said, in the Council there would be equal representation of high-caste
Hindus and Muslims. Other minorities including low-caste Hindus, Shudders and
Sikhs would be given representation in the Council.
Statement 3: It proposed for a future constitution of India, not its partition.

(C) Insights Active Learning. | All rights reserved.

www.insightsias.com

34

TEST - 12

Q Source: CSP 2007

IA
S

49 Which of the following statements about the species Slow Loris is INCORRECT?
A. They can be found in Southeast Asia.
B. They have a toxic bite.
C. They are covered under the Convention on International Trade in Endangered
Species of Wild Fauna and Flora (CITES).
D. None of the above
User Answer :
Correct Answer : D
Answer Justification :

TS

Justification:Option A: They are found in Southeast Asia and bordering areas, they
range from Bangladesh and Northeast India in the west to the Sulu Archipelago in
the Philippines in the east, and from Yunnan province in China in the north to the
island of Java in the south.

Option B: Slow lorises have a toxic bite, a trait rare among mammals and unique to
lorisid primates. The toxin is obtained by licking a gland on their arm, and the
secretion is activated by mixing with saliva. Their toxic bite is a deterrent to
predators, and the toxin is also applied to the fur during grooming as a form of
protection for their infants.

SI

Option C: Each of the slow loris species that had been identified prior to 2012 is
listed as either "Vulnerable" or "Endangered" on the IUCN Red List.
Q Source: CSP 2007

IN

50 Consider the following with regard to the Council of Scientific and Industrial Research
(CSIR).
1. President of CSIR is the Union Minister of Science and Technology.
2. Union Minister of Finance and Minister of Commerce and Industry are its members.
3. It works as an attached office under the Ministry of Science and Technology.
Select the correct answer using the codes below.
A. 1 and 2 only
B. 2 only
C. 1 and 3 only
D. None of the above
User Answer :
Correct Answer : B

(C) Insights Active Learning. | All rights reserved.

www.insightsias.com

35

TEST - 12

Answer Justification :
Justification:Statement 1: Prime Minister is the President of CSIR. Minister for
S&T is its Vice- President.
Statement 2: Here is the list of members:

IA
S

http://www.csir.res.in/External/Heads/aboutcsir/org_structure_files/members.pdf
Statement 3: Although it is mainly funded by the Ministry of Science and
Technology, it operates as an autonomous body registered under the Registration of
Societies Act of 1860.

Q Source: CSP 2006

TS

Council of Scientific and Industrial Research (CSIR), established in 1942, is the


largest research and development (R&D) organisation in India.

SI

51 Seventh Schedule of the Constitution of India deals with


A. Protection of the rights of tribals in hilly and forest areas
B. Powers of the Executive branch of the government
C. Allowances of the President, Vice-President and Supreme Court Judges
D. Division of powers between the Union and the States
User Answer :
Correct Answer : D
Answer Justification :

IN

Learning:The legislative section is divided into three lists: Union List, State List
and Concurrent List. Unlike the federal governments of the United States,
Switzerland or Australia, residual powers remain with the Union Government, as
with the Canadian federal government
The Union List or List-I is a list of 100 items given in Seventh Schedule in the
Constitution of India on which Parliament has exclusive power to legislate.
You may read
thishttp://indianexpress.com/article/opinion/columns/art-of-the-state-2/
Q Source:CSP 2006

52 Fiscal Responsibility and Budget Management Act (FRBMA) exhorts the Union

(C) Insights Active Learning. | All rights reserved.

www.insightsias.com

36

TEST - 12

IA
S

Government to
1. Stop borrowing from markets to finance budget deficit
2. Allow State Governments to borrow from International markets
3. Reduce the size of the Foreign Exchange Reserves to reduce Budget Deficit
Select the correct answer using the codes below.
A. 1 and 2 only
B. 2 and 3 only
C. 2 only
D. None of the above
User Answer :
Correct Answer : D
Answer Justification :

TS

Justification: Statement 1: The Act provided that the Central Government shall not
borrow from the Reserve Bank of India (RBI) except under exceptional
circumstances where there is temporary shortage of cash in particular financial year.
It was not about restricting market borrowing.
Statement 2: It has no such provision.

Statement 3: Foreign Exchange Reserves related budget financing is not covered by


the Act. Moreover, reducing Forex reserves and reducing budget deficit are two
different things.

SI

We will be covering more questions on the Act since the 14th Finance Commission
has recommended replacing the Act with a new one.
Q Source:CSP 2006

IN

53 What is the difference between the CMYK and RGB colour modes that are used in
graphic display and printing?
1. RGB model is a colour additive model, whereas CMYK is a subtractive model.
2. Image made in CMYK mode cannot be printed, whereas under RGB mode it can be
printed.
Which of the above is/are correct?
A. 1 only
B. 2 only
C. Both 1 and 2
D. None
User Answer :
Correct Answer : A

(C) Insights Active Learning. | All rights reserved.

www.insightsias.com

37

TEST - 12

Answer Justification :
Justification & Learning:RGB stands for the colors red, green, and blue, the colors
widely recognized in design fields as the primary colors. The RGB model is known
as an additive model, where colors are added together to make up what we see on
the screen.

Q Source:CSP 2006

TS

IA
S

Basically, pixels on a television set or computer monitor create tiny pixels


that, if viewed under a magnifying glass, are one of those three colors. Light
is projected through them, blending the colors on the eye's retina to create the
desired colors.
CMYK, on the other hand, stands for the colors cyan, magenta, yellow, and
black. CMYK is a subtractive model. This gets a bit complicated, but the idea
with subtractive models like CMYK is that colors from the spectrum are
subtracted from natural white light into pigments or dyes. These pigments,
then, are printed onto paper in tiny little cyan, magenta, yellow, and black
dots.
If you were to take a magnifying glass to a magazine cover, for example, you
would see that the main image is really just a bunch of dots spread out, some
closer than others, to appear like the colors we want.
Images made in both colour modes can be printed, but the printers generally
only use the CMYK mode to print.

IN

SI

54 Naqqar khana was


A. A drum house from where ceremonial music was played in Mughal palaces
B. A prayer hall indicating the direction of Mecca
C. The discussion hall for secret courtly affairs in the Mughal palaces
D. Made to host foreign dignitaries in Mughal complexes
User Answer :
Correct Answer : A
Answer Justification :
Learning:It was usually situated over the gate.
They are a distinct sign of Mughal architecture and were constructed under areas
their influence in India, Pakistan and nearby countries.
The pavilion named Naubat Khana in Red Fort in Delhi is near the entrance on the
eastern side of the ten pillars lane, next to a different pavilion where royal

(C) Insights Active Learning. | All rights reserved.

www.insightsias.com

38

TEST - 12

TS

IA
S

palanquins and other paraphernalia were placed. It housed 18 kinds of musical


instruments that used to form part of the royal entourage.

Q Source: Page 128: Chapter 8: 11th NCERT: An Introduction to Indian Arts

IN

SI

55 Consider the following about the Central Electricity Regulatory Commission (CERC).
1. It is a statutory body functioning with quasi-judicial status.
2. It regulates the tariff of only state owned power Generating companies.
3. It approves and executes major government investment projects in electricity
industry.
Select the correct answer using the codes below.
A. 1 and 2 only
B. 2 and 3 only
C. 1 only
D. 2 only
User Answer :
Correct Answer : C
Answer Justification :
Justification:Statement 1: a key regulator of power sector in India, is a statutory

(C) Insights Active Learning. | All rights reserved.

www.insightsias.com

39

TEST - 12

body functioning with quasi-judicial status under sec ' 76 of the Electricity Act
2003.
Statement 2: CERC was instituted primarily to regulate the tariff of Power
Generating companies owned or controlled by the government of India, and any
other generating company which has a composite scheme for

IA
S

power generation and interstate transmission of energy, including tariffs of


generating companies. So, 2 would be wrong.

Statement 3: It does not approve and execute, but advise in promoting investment in
the electricity sector in the economy. So, 3 would be wrong.

TS

Q Source: CSP 2006: On Appellate Tribunal for Electricity

SI

56 Consider the following about Abdul Hamid Lahori.


1. He was the court historian of Shah Jahan.
2. He wrote the book Padshahnama.
3. He was admitted to the Mansabdari system.
Select the correct answer using the codes below.
A. 1 and 2 only
B. 2 and 3 only
C. 1 and 3 only
D. 1, 2 and 3
User Answer :
Correct Answer : A
Answer Justification :

IN

Learning:Abdul Hamid Lahori was a traveller and historian during the period of
Shah Jahan who later became a court historian of Shah Jahan.
He wrote the book Padshahnama also referred as Badshahnama, about the reign of
Shah Jahan. He has described Shah Jahan's life and activities during the first twenty
years of his reign in this book in great detail.
In his own preface to the text, Lahori mentions that he called from his retirement in
Patna to write official history as the Emperor wanted someone who could emulate
the style of Akbarnama of Abul Fazl which he too greatly admired.
Q Source:CSP 2006

(C) Insights Active Learning. | All rights reserved.

www.insightsias.com

40

TEST - 12

TS

IA
S

57 Consider the following statements about Madam Bhikaji Cama.


1. She co-founded the Paris Indian Society.
2. She has once served as the president of the British Committee of the Indian National
Congress (INC).
3. She unfurled the National Flag at the International Socialist Conference in Paris in
1907.
4. She has served as the Secretary General of the Indian Home Rule Society.
Select the correct answer using the codes below.
A. 1 and 3 only
B. 2, 3 and 4 only
C. 3 only
D. 1, 2 and 4 only
User Answer :
Correct Answer : A
Answer Justification :
Justification:Statement 1: She co-founded the Paris Indian Society together with
Singh Rewabhai Rana and Munchershah Burjorji Godre.

Statement 2 and 4: She served as the private secretary of Dadabhai Naoroji, the
president of the British Committee of the Indian National Congress. Together with
Naoroji and Singh Rewabhai Rana, Cama supported the founding of Varma's Indian
Home Rule Society in 1905.

SI

Statement 3: In her appeal for human rights, equality and for autonomy from Great
Britain, she unfurled what she called the "Flag of Indian Independence"
Q Source:CSP 2006

IN

58 In the Nagara (North Indian temple style)


1. Elaborate boundary walls or gateways are usually not constructed.
2. The garbhagriha is always located directly under the tallest tower.
Select the correct answer using the codes below.
A. 1 only
B. 2 only
C. Both 1 and 2
D. None
User Answer :
Correct Answer : C
Answer Justification :

(C) Insights Active Learning. | All rights reserved.

www.insightsias.com

41

TEST - 12

Justification:In North India it is common for an entire temple to be built on a stone


platform with steps leading up to it.
Further, unlike in South India it does not usually have elaborate boundary walls or
gateways. While the earliest temples had just one tower, or shikhara, later temples
had several. The garbhagriha is always located directly under the tallest tower.

IA
S

There are many subdivisions of nagara temples depending on the shape of the
shikhara.
Q Source: Page 71: Chapter 6: 11th NCERT: An Introduction to Indian Arts

SI

TS

59 The central government is promoting Nagpur in Maharashtra as the tiger capital. It is for
which of the following reason(s)?
1. To promote tiger conservation as their numbers have declined to alarming levels in
the State in the past few decades.
2. There are many notified tiger parks and reserves in its close proximity.
Select the correct answer using the codes below.
A. 1 only
B. 2 only
C. Both 1 and 2
D. None
User Answer :
Correct Answer : B
Answer Justification :

IN

Justification:Statement 1: As per a official statement, Maharashtra has done


exceptionally well in the growth of tigers- which has prompted the government to
promote Nagpur, which is the central most city of the country. So, 1 is wrong.
Statement 2: It is promoted as the tiger capital of the world given the city's closeness
with several tiger reserves and its infrastructure that can support a large tourist
industry.
Maharashtra has a tiger population of 169 according to latest census. Nagpur and its
nearby reserves in Maharashtra have 148 of these. In addition, the city was close to
many tiger reserves in Madhya Pradesh like Kanha, Pench, and Satpuda.
Q Source: CAPF 2011

(C) Insights Active Learning. | All rights reserved.

www.insightsias.com

42

TEST - 12

TS

Justification: Parliament may by law

IA
S

60 Who/Which of the following is empowered to alter the name of a State?


1. Governor of the State concerned
2. State Legislative Assembly
3. Parliament
Select the correct answer using the codes below.
A. 1 and 2 only
B. 1 and 3 only
C. 2 only
D. 3 only
User Answer :
Correct Answer : D
Answer Justification :

form a new State by separation of territory from any State or by uniting two
or more States or parts of States or by uniting any territory to a part of any
State;
increase the area of any State;
diminish the area of any State;
alter the boundaries of any State;
alter the name of any State; Provided that no Bill for the purpose shall be
introduced in either House of Parliament except on the recommendation of
the President

SI

Q Source:CAPF 2011

IN

61 The hydraulic brake is an arrangement of braking mechanism which uses brake fluid to
transfer
1. Pressure from the controlling mechanism to the braking mechanism
2. Momentum from the angular belts to the wheel base
Select the correct answer using the codes below.
A. 1 only
B. 2 only
C. Both 1 and 2
D. None
User Answer :
Correct Answer : A
Answer Justification :

(C) Insights Active Learning. | All rights reserved.

www.insightsias.com

43

TEST - 12

Justification: Hydraulic brakes transfer energy to stop an object, normally a


rotating axle. In a very simple brake system, with just two cylinders and a disc
brake, the cylinders could be connected via tubes, with a piston inside the cylinders.

IA
S

The cylinders and tubes are filled with incompressible oil. The two cylinders have
the same volume, but different diameters, and thus different cross-section areas. The
one with the smallest diameter is called the master cylinder.
The spinning disc brake will be placed down at[clarification needed] the piston with
the larger cross-section. Suppose the diameter of the master
cylinder is half the diameter of the slave cylinder, so the master cylinder has a crosssection four times smaller.

TS

Now, if the piston in the master cylinder is pushed down 40 mm, with 10 newtons
(N) of force, the slave piston will then move 10 mm, with a force of 40 N.
Q Source:CSP 2006

IN

SI

62 Which of the following was/were the objective(s) of the Theosophical Society?


1. To investigate the unexplained laws of nature and the powers latent in man
2. To encourage the study of comparative religion
Select the correct answer using the codes below.
A. 1 only
B. 2 only
C. Both 1 and 2
D. None
User Answer :
Correct Answer : C
Answer Justification :
Justification & Learning: After several iterations the Society's objectives evolved
to be:
To form a nucleus of the universal brotherhood of humanity without
distinction of race, creed, sex, caste, or colour.
To encourage the study of comparative religion, philosophy, and science and
rational enquiry into the unexplained

Q Source:SET A: Q92: CAPF 2011

(C) Insights Active Learning. | All rights reserved.

www.insightsias.com

44

TEST - 12

IA
S

63 Consider the following about the Red panda.


1. It is native to the eastern Himalayas and south-western China.
2. It has been classified as endangered by the IUCN.
3. It is highly active during the day.
Select the correct answer using the codes below.
A. 1 only
B. 1 and 3 only
C. 2 and 3 only
D. 1 and 2 only
User Answer :
Correct Answer : D
Answer Justification :

TS

Justification:Statement 1: Distribution of the red panda is disjointed, with two


extant subspecies:

Western red panda lives in the western part of its range, in Nepal, Assam,
Sikkim, and Bhutan.
Styan's red panda lives in the east-north-eastern part of its range, in southern
China and northern Burma.

Statement 2: It is also covered under CITES. Habitat loss, poaching and hunting are
the biggest threats to red panda.

SI

Statement 3: It is a solitary animal, mainly active from dusk to dawn, and is largely
sedentary during the day.
Q Source: SET A: Q150: CSP 2009

IN

64 The non-permanent members of the United Nations Security Council (UNSC) are
A. Elected for two-year terms by the General Assembly
B. Nominated by the permanent members of UNSC
C. Appointed by the UN Secretary General
D. Selected on rotation basis from the members of the General Assembly
User Answer :
Correct Answer : A
Answer Justification :
Learning:The Council is composed of 15 Members:
five permanent members: China, France, Russian Federation, the United Kingdom,

(C) Insights Active Learning. | All rights reserved.

www.insightsias.com

45

TEST - 12

and the United States, and ten non-permanent members.


More than 60 United Nations Member States have never been Members of the
Security Council.

IA
S

A State which is a Member of the United Nations but not of the Security Council
may participate, without a vote, in its discussions when the Council considers that
that country's interests are affected.
Both Members and non-members of the United Nations, if they are parties to a
dispute being considered by the Council, may be invited to take part, without a vote,
in the Council's discussions; the Council sets the conditions for participation by a
non-member State.

TS

Q Source: SET A: Improvisation: Q91: CSP 2009

IN

SI

65 Consider the following statements about Neutrinos.


1. They can be found only under the earth's crust.
2. Human body cannot tolerate exposure to Neutrinos.
3. They are used to trigger fission in nuclear reactors.
4. They are completely massless.
5. They have the ability to carry information from one part of the Universe to the other.
6. They cannot reach electrically neutral states.
Choose the correct answer using the codes below.
A. 1 and 2 only
B. 5 only
C. 3 and 4 only
D. 2, 4 and 6 only
User Answer :
Correct Answer : B
Answer Justification :
Justification:Neutrinos are the information bearers of the universe ' which are
almost never lost in their path. India's effort in studying neutrinos at INO may help
us unravel the deepest mystery of the universe ' why there is more matter than
antimatter in the universe. So, 5 is correct.
Neutrinos are the least harmful of all elementary particles (so, 1 is wrong), as
they almost never react with solid bodies. So, 2 is incorrect.
Also, people tend to confuse the 'neutrino' for the 'neutron'. This has also led
to the confusion that neutrinos can be weaponised, which is far from the truth.

(C) Insights Active Learning. | All rights reserved.

www.insightsias.com

46

TEST - 12

Q Source:SET A: Q40: CAPF 2012

IA
S

They are not used to trigger fission reactions, but are instead produced as a
result of the reactions. So, 3 is incorrect.
Neutrinos have mass and as shown recently, they change their masses too. So,
4 is incorrect.
Neutrinos, named as such because they are electrically neutral, are leptons,
and so are not affected by the strong force either. So, 6 is wrong.

TS

66 Consider the following statements about the Malabar rebellion.


1. It was an armed uprising in 1921 against British authority and Hindu landlords.
2. It began as a reaction against a heavy-handed crackdown on the Khilafat Movement
by the British authorities in Kerala.
3. The "Wagon tragedy" was related to this rebellion.
Select the correct answer using the codes below.
A. 1 only
B. 2 and 3 only
C. 1 and 3 only
D. 1, 2 and 3
User Answer :
Correct Answer : D
Answer Justification :

SI

Learning:It was launched by Mappila Muslims and was the culmination of a series
of Mappila revolts that recurred throughout the 19th century and early 20th century.
The Mappilas attacked and took control of police stations, British government
offices, courts and government treasuries.

IN

In the later stages of the uprising, Mappilas committed several atrocities against the
Hindu community, who they accused of helping the police to suppress their
rebellion.
Q Source:SET A: Q51: CAPF 2012

67 Public Choice Theory stresses on


1. Single monolithic institution should deliver all public services.
2. Only those services should be given for which a citizen can pay.
3. Bureaucracy should be strengthened and there should be a single centre of power.
Select the correct answer using the codes below.

(C) Insights Active Learning. | All rights reserved.

www.insightsias.com

47

TEST - 12

IA
S

A. 1 and 2 only
B. 2 only
C. 3 only
D. None of the above
User Answer :
Correct Answer : D
Answer Justification :
Learning:All statements go against the philosophy of Public Choice Theory.
Public Choice Approach Theory strictly opposes the bureaucratic model of
administration.

TS

As per the theory, 'Perfection in the hierarchical ordering of a professionally trained


public service accountable to a single centre of power will reduce the capability of a
larger administrative system to respond to diverse preferences among citizens for
many different public goods and services and cope with diverse environmental
conditions."

It supports institutional pluralism and says that if market forces are allowed to
operate freely, than there are full chances that they will function with much higher
efficiency than the Government. Presence of market forces will reduce the
unnecessarily inflated size of the Government /Bureaucracy and make the delivery
of goods and services to the customers more transparent, cost-effective, simpler and
economical.

SI

Q Source:Improvisation: SET A: Q65: CAPF 2012

IN

68 What is Byssinosis?
A. An occupational lung disease caused by exposure to cotton dust in
inadequately ventilated working environments
B. A disease prevalent in those working in deep coal mines that are cutoff from
sunlight
C. A nutrient deficiency among asbestos miners and those who work with
asbestos insulation
D. A symptom common among persons working in the information technology
sector
User Answer :
Correct Answer : A
Answer Justification :

(C) Insights Active Learning. | All rights reserved.

www.insightsias.com

48

TEST - 12

Learning: It is also called "brown lung disease" or "Monday fever".


Byssinosis commonly occurs in workers who are employed in yarn and fabric
manufacture industries. It is now thought that the cotton dust directly causes the
disease and some believe that the causative agents are endotoxins that come from
the cell walls of gram negative bacteria that grow on the cotton.

IA
S

Although bacterial endotoxin is a likely cause, the absence of similar symptoms in


workers in other industries exposed to endotoxins makes this uncertain.
Q Source: Improvisation: SET A: Q73: CAPF 2012

TS

69 In India, wild animals can be declared as vermin if


1. They have become dangerous to human life or property
2. Make the ecology aesthetically unclean
3. Become disabled or diseased as to be beyond recovery
Select the correct answer using the codes below.
A. 1 and 2 only
B. 2 and 3 only
C. 1 and 3 only
D. 1, 2 and 3
User Answer :
Correct Answer : C
Answer Justification :

SI

Learning:The Union Ministry of Environment, Forest and Climate Change


(MoEFCC) has declared Wild Pig as vermin in Uttarakhand for a year.

IN

This decision will allow state forest authorities and people to carry out an
extermination (hunting/poaching) of wild pigs outside the reserve forest on a large
scale.
To do so, they will no longer require permission from the forest or wildlife officials
and thus their actions cannot attract penal provisions of the WPA.
It will seek to balance local population of the wild pigs to mitigate the damage to
human life, crops and other properties in the state for ensuring conservation of
wildlife in forests.
Q Source:
http://www.livemint.com/Politics/TSea6J0X9dNMS2eibTMW9M/Wild-pigs-declar

(C) Insights Active Learning. | All rights reserved.

www.insightsias.com

49

TEST - 12

ed-vermin-in-Uttarakhand-for-a-year.html

TS

IA
S

70 India and United Arab Emirates (UAE) have signed bilateral agreements related to
cooperation in which of the following?
1. Exploration and use of Outer Space for Peaceful Purposes
2. Bilateral Currency Swap Arrangement
3. Technical Cooperation in Cyber Space and Combating Cyber Crime
Select the correct answer using the codes below.
A. 1 and 2 only
B. 2 only
C. 1 and 3 only
D. 1, 2 and 3
User Answer :
Correct Answer : D
Answer Justification :
Learning: Some of the agreements and their details:

IN

SI

MoU in the Exploration and use of Outer Space for Peaceful Purposes:
Framework for cooperation in space technology, science and applications
including satellite communication, remote sensing and satellite based
navigation.
MoU on Bilateral Currency Swap Arrangement: Promotes currency swap
arrangement between the RBI & Central Bank of UAE for bilateral financial
relations.
Both sides also signed a crucial MoU on establishing a framework for
facilitating the participation of UAE institutional investors in Infrastructure
Investments in India.
This would see investments from UAE's sovereign wealth fund. However, the
quantum of investment was yet to be worked out.

Q
Source:http://www.thehindubusinessline.com/money-and-banking/india-uae-sign-p
act-for-bilateral-currency-swap/article8224369.ece

71 Apart from the Permanent members of the UNSC, who/which among the following
have the right to use veto power to stall key decisions in the United Nations (UN)?
1. Founding members of the UN
2. UN Secretary General
3. Countries that contribute substantially to the UN Budget

(C) Insights Active Learning. | All rights reserved.

www.insightsias.com

50

TEST - 12

IA
S

Select the correct answer using the codes below.


A. 1 and 2 only
B. 1 and 3 only
C. 2 only
D. None of the above
User Answer :
Correct Answer : D
Answer Justification :

Justification:Power of veto" refers to the veto power wielded solely by the five
permanent members of the United Nations Security Council (China, France, Russia,
United Kingdom, and United States), enabling them to prevent the adoption of any
"substantive" resolution, as well as decide which issues fall under "substantive" title.

TS

This de facto control over the UN Security Council by the five governments is seen
by critics, since its creation in 1945, as the most undemocratic character of the UN.

The veto is exercised when any permanent member'the so-called "P5"'casts a


"negative" vote on a "substantive" draft resolution. Abstention or absence from the
vote by a permanent member does not prevent a draft resolution from being adopted.

Q Source:Chapter 6: 12th NCERT: Contemporary World Politics

IN

SI

72 Indian Handloom Tag has been given to which of the following?


1. Pedana Fabric
2. Chettinad Cotton
3. Ronda Wool
Select the correct answer using the codes below.
A. 1 and 2 only
B. 2 only
C. 2 and 3 only
D. 1, 2 and 3
User Answer :
Correct Answer : A
Answer Justification :
Learning: The Union textile Ministry has bestowed the Indian Handloom Brand tag
on Chettinad cotton saris for its unique designs and identity. This tag was bestowed
under the India Handloom Brand Scheme after checking its various quality
parameters stipulated by the Standard Operating Procedure (SOP). The Ministry also
has offered a logo to use the brand for marketing the product. It will also enable

(C) Insights Active Learning. | All rights reserved.

www.insightsias.com

51

TEST - 12

weavers of these saris to promote the material in the international market.


Chettinad cotton saree is also known as 'kandaangi'. It is the traditional sari of the
Chettinad region in Sivaganga district of Tamil Nadu. It is characterised by its silklike texture, contrasting border and body colours with intricate designs along the
border.

IA
S

Q
Source:http://www.thehindu.com/news/national/andhra-pradesh/india-handloom-br
and-tag-for-pedana-fabric/article7743387.ece
http://www.thehindu.com/news/national/tamil-nadu/chettinad-cotton-saree-wins-ind
ia-handloom-tag/article8204692.ece

SI

TS

73 Scientists recently successfully converted paper waste into green nontoxic cellulose
aerogels. Aerogels find potential application in
1. Improving drug delivery
2. Oil spill cleaning
3. Heat insulation materials for buildings
Select the correct answer using the codes below.
A. 1 and 2 only
B. 2 and 3 only
C. 3 only
D. 1, 2 and 3
User Answer :
Correct Answer : D
Answer Justification :

IN

Justification:Statement 1: It can be used in a drug delivery system owing to its


biocompatibility. Due to its high surface area and porous structure, drugs can be
adsorbed from supercritical CO2. The release rate of the drugs can be tailored by
varying the properties of the aerogel.
Statement 2: It can be used as a chemical adsorber for cleaning up spills.
Statement 3: Aerogel is a synthetic porous ultralight material derived from a gel, in
which the liquid component of the gel has been replaced with a gas. The result is a
solid with extremely low density and low thermal conductivity. It is nearly 98% air.
So, it fits as a heat insulation material. 3 is correct.
Q

(C) Insights Active Learning. | All rights reserved.

www.insightsias.com

52

TEST - 12

Source:http://www.thehindu.com/todays-paper/tp-national/paper-waste-turned-intonontoxic-aerogel/article8207322.ece

IA
S

74 The 12th edition of South Asian Games (SAG) was conducted in Guwahati which saw
participation of foreign nationals. For India, hosting such a game amounts to
A. Exports
B. Imports
C. Foreign Arbitrage
D. Foreign Institutional Investment
User Answer :
Correct Answer : A
Answer Justification :

TS

Justification:Instead of consuming Indian production from abroad, these participant


foreign nationals would consume it here in India.
In a way, it amounts to increased exports from India.

It cannot be Foreign Institutional Investment as that is an investment in the country's


security markets from abroad. A visit to India is not needed.

Q Source:
http://indianexpress.com/article/cities/chandigarh/south-asian-games-in-guwahati-ch
andigarh-swimmer-wins-first-medal-in-international-competition/

IN

SI

75 Which of the following statements is INCORRECT about Asia Pacific Economic


Cooperation (APEC)?
A. All its members are from the Northern Hemisphere.
B. It promotes free trade throughout the Asia-Pacific region.
C. India is not its member country.
D. It was established before the World Trade Organization (WTO).
User Answer :
Correct Answer : A
Answer Justification :
Justification:Australia, Chile, New Zealand are from the Southern Hemisphere. So,
(a) is wrong.
It was established in 1989 (WTO in 1995) in response to the growing
interdependence of Asia-Pacific economies and the advent of regional trade blocs in

(C) Insights Active Learning. | All rights reserved.

www.insightsias.com

53

TEST - 12

other parts of the world to establish new markets for agricultural products and raw
materials beyond Europe. It promotes free trade between the rim countries of Asiapacific. So, (b) and (d) are correct.
India has requested for APEC membership. So, (c) is correct.

IA
S

Q Source:SET A: Improvisation: Q3: CSP 2009

TS

76 Why is the PQP certification granted by the World Health Organization (WHO)
important for Indian pharmaceutical companies?
A. It allows them to do business in nations with frequent epidemic outbreaks
B. It removes restrictions on export of generic drugs to developing countries
C. It allows these companies to claim domestic patents globally
D. It helps them participate in the global immunisation programme which is
facilitated through international procurement agencies like UNICEF
User Answer :
Correct Answer : D
Answer Justification :

Learning: Every year, billions of US dollars worth of medicines are purchased by


or through international procurement agencies - such as UNICEF, the Global Fund
to Fight AIDS, Tuberculosis and Malaria, and UNITAID - for distribution in
resource-limited countries.

IN

SI

The WHO Prequalification of Medicines Programme (PQP) helps ensure that


medicines supplied by procurement agencies meet acceptable standards of
quality, safety and efficacy.
At the end of 2012, the WHO List of Prequalified Medicinal Products
contained 316 medicines for priority diseases.
WHO's list of prequalified medicinal products is used by international
procurement agencies and increasingly by countries to guide bulk purchasing
of medicines.
PQP also prequalifies active pharmaceutical ingredients and quality control
laboratories.

Q Source:
http://www.thehindu.com/business/who-certifies-vaccine-maker-greensignal-bio/arti
cle8194263.ece
77 Countries that are party to the Trans-Pacific Partnership (TPP) Agreement are

(C) Insights Active Learning. | All rights reserved.

www.insightsias.com

54

TEST - 12

Australia
China
Japan
Germany
Chile
Select the correct answer using the codes below.
A. 1, 3 and 5 only
B. 1, 2 and 4 only
C. 3, 4 and 5 only
D. 1, 2, 3, 4 and 5
User Answer :
Correct Answer : A
Answer Justification :

IA
S

1.
2.
3.
4.
5.

TS

Justification & Learning:All 12 Pacific Rim countries have recently signed TransPacific Partnership (TPP) agreement in New Zealand making it one of the biggest
trade deals in history.

These 12 countries are: Singapore, Malaysia, Vietnam, Brunei Darussalam, Japan,


Canada, United States, Mexico, Peru, Chile, Australia and New Zealand.

TPP is Free Trade Area and these member countries across the Pacific Rim are
home to 800 million people and account for 40% of global trade.
Objectives of TPP are:

IN

SI

(i) Lower trade barriers such as tariffs.


(ii) Establish a common framework for intellectual property
(iii) Enforce standards for labour law and environmental law
(iv) Establish an investor-state dispute settlement mechanism.

Q Source:
http://www.thehindu.com/business/12-nations-sign-transpacific-partnership-trade-de
al/article8191992.ece

78 The Union Government has constituted the Tax Policy Council (TPC) to streamline the
taxation policy and administration on the recommendation of the Tax Administration
Reform Commission (TARC). Apart from selected Union Ministers and Secretaries, TPC
member(s) is/are
1. NITI Aayog Chairman
2. Chief Economic Advisor

(C) Insights Active Learning. | All rights reserved.

www.insightsias.com

55

3. RBI Governor
Select the correct answer using the codes below.
A. 1 and 2 only
B. 2 only
C. 2 and 3 only
D. None of the above
User Answer :
Correct Answer : B
Answer Justification :

IA
S

TEST - 12

TS

Justification & Learning:The council will be headed by Union Finance Minister. It


shall have 9 members - Minister of State for Finance, Commerce Minister, NITI
Aayog Vice-Chairman (and not Chairman who is the Prime Minister), Chief
Economic Advisor and Finance Secretary.
It would also have secretaries from the department of Revenue, DEA, DIPP and
Ministry of Commerce.

Another Tax Policy Research Unit (TPRU) will be headed by Revenue Secretary. It
will carry out studies on various topics of fiscal and tax policies. It will assist the
TPC in taking appropriate policy decisions and shall prepare tax proposal and
analysis of legislative intent.

SI

Q Source:
http://www.thehindubusinessline.com/economy/govt-sets-up-two-new-tax-committe
es-to-bring-coherence-in-policies/article8184415.ece

IN

79 Albedo is the fraction of solar energy which is


A. Reflected from the Earth back into space
B. Absorbed by autotrophic organisms
C. Directly incident on Goldilocks zones
D. Captured by clouds
User Answer :
Correct Answer : A
Answer Justification :
Learning: It is a measure of the reflectivity of the earth's surface. Ice, especially
with snow on top of it, has a high albedo: most sunlight hitting the surface bounces
back towards space.
Water is much more absorbent and less reflective. So, if there is a lot of water, more

(C) Insights Active Learning. | All rights reserved.

www.insightsias.com

56

TEST - 12

solar radiation is absorbed by the ocean than when ice dominates.


Albedo is not important at high latitudes in winter: there is hardly any incoming
sunlight to worry about. It becomes important in spring and summer when the
radiation entering through leads can greatly increase the melt rate of the sea ice.

IA
S

Q Source: CSP 2008

TS

80 The Council for Scientific and Industrial Research (CSIR) has launched BGR-34. It is
the country's first
A. Fast Breeder Thorium Reactor
B. Anti-diabetic ayurvedic drug
C. Centralized database system for tackling cyber crimes
D. Molecular distillation technology to process shale gas from rocks
User Answer :
Correct Answer : B
Answer Justification :

Learning: BGR-34 stands for Blood Glucost Regulator and offers effective, safe,
patient-friendly solution for managing type two Diabetes.

The drug has been jointly developed by National Botanical Research Institute
(NBRI) and Central Institute for Medicinal and Aromatic Plants (CIMAP) and CSIR
institutes based in Lucknow.

SI

It contains 34 identified active phyto-constitutes from herbal resources and works by


controlling blood sugar and limiting the harmful effects of other modern drugs.

IN

Q
Source:http://www.huffingtonpost.in/2015/10/26/rs-5-herbal-drug-diabetes_n_8387
388.html
http://www.thehindu.com/todays-paper/tp-national/tp-kerala/ayurvedic-antidiabeticdrug-launched/article8191078.ece

81 Which among the following is the world's largest producer of Uranium?


A. Canada
B. Australia
C. Russia
D. Kazakhstan
User Answer :

(C) Insights Active Learning. | All rights reserved.

www.insightsias.com

57

TEST - 12

Correct Answer : D
Answer Justification :
Learning:Canada was the world's largest uranium producer for many years,
accounting for about 22% of world output, but in 2009 was overtaken by
Kazakhstan,which now accounts for nearly 37% of world production.

IA
S

Production comes mainly from the McArthur River and Cigar Lake mines in
northern Saskatchewan province, which are the largest and highest-grade in the
world.
Australia is the third largest accounting for nearly 10%; Russia nearly 5%.

TS

Q Source: SET A: Improvisation: Q30: CSP 2009

SI

82 Consider the following about the Bharhut Stupa.


1. The Shungas improvised the art work at Bharhut.
2. Jataka tales have been depicted on its railings.
3. Queen Maya's dream has been depicted at Bharhut.
Select the correct answer using the codes below.
A. 1 and 2 only
B. 2 and 3 only
C. 1 and 3 only
D. 1, 2 and 3
User Answer :
Correct Answer : D
Answer Justification :

IN

Justification: Statement 1: The Bharhut stupa (in MP) may have been first built by
the Maurya king Ashoka in the 3rd century BCE, but many works of art were
apparently added during the Shunga period, with many friezes from the 2nd century
BCE.
Statement 2: They contain numerous birth stories of the Buddha's previous lives, or
Jataka tales. Many of them are in the shape of large, round medallions.
Statement 3: Queen Maya's dream, preceding the birth of the Buddha, is also a
major theme on the railing of the Bharhut "stupa".

(C) Insights Active Learning. | All rights reserved.

www.insightsias.com

58

TS

IA
S

TEST - 12

Q Source: Page 28: Chapter 4: 11th NCERT: An Introduction to Indian Arts

IN

SI

83 Dhammacakkappavattana Sutta is
A. A set of secular codes for Buddhist monks
B. Considered to be a record of the first teaching given by Gautama Buddha
after he attained enlightenment.
C. The compilation of philosophical work on Buddhist teachings by religious
scholars
D. The last teaching given by the Buddha before his Mahaparinirvana
User Answer :
Correct Answer : B
Answer Justification :
Learning: The Setting in Motion of the Dharmachakra Sutra or Promulgation of the
Law Sutra) is a Buddhist text that is considered to be a record of the first teaching
given by Gautama Buddha after he attained enlightenment.
According to tradition, the Buddha gave this teaching in Sarnath, India, to the "five
ascetics", his former companions with whom he had spent six years practicing
austerities.
The main topic of this sutra is the Four Noble Truths, which are the central
teachings of Buddhism that provide a unifying theme, or conceptual framework, for

(C) Insights Active Learning. | All rights reserved.

www.insightsias.com

59

TEST - 12

all of Buddhist thought. This sutra also introduces the Buddhist concepts of the
Middle Way, impermanence, and dependent origination.
Q Source: Page 26: Chapter 3: 11th NCERT: An Introduction to Indian Arts

IA
S

84 Which among the following statements about South Asia is correct?


A. The institution of monarchy has been completely abolished in South Asia.
B. Other than Sri Lanka, there is no island state in South Asia.
C. The combined Nominal GDP of South Asia is less than that of China.
D. All countries fall under the same time zone.
User Answer :
Correct Answer : C
Answer Justification :

TS

Justification: Bhutan is a constitutional monarchy with a parliamentary form of


government. So, (a) is wrong.
Maldives is also a part of South Asia, which is an island state. So, (b) is wrong.

The combined Nominal GDP of South Asia is nearly $ 3 trillion whereas that of
China is around $11.3 trillion! So, (c) is correct.

The time zones cary from UTC +4:30 to UTC+6:00. So, (d) is incorrect.

SI

Q Source: Chapter 5: 12th NCERT: Contemporary World Politics

IN

85 Consider the following about the Shanti Swarup Bhatnagar Prize.


1. The purpose of the prize is to recognize outstanding work in science and technology.
2. It can be awarded to any Indian or foreign citizen.
3. The prize is distributed by the Prime Minister of India.
4. The awardee is bound to give a lecture in the area of the award.
Select the correct answer using the codes below.
A. 1 only
B. 2 and 4 only
C. 1, 3 and 4 only
D. 1, 2, 3 and 4
User Answer :
Correct Answer : C
Answer Justification :

(C) Insights Active Learning. | All rights reserved.

www.insightsias.com

60

TEST - 12

Justification: Statement 1: It is a science award in India given annually by the


Council of Scientific and Industrial Research (CSIR) for notable and outstanding
research, applied or fundamental, in biology, chemistry, environmental science,
engineering, mathematics, medicine and Physics.

IA
S

Statement 2: Any citizen of India engaged in research in any field of science and
technology up to the age of 45 years is eligible for the prize.
Statement 3 and 4: The names of the recipients are traditionally declared by the
Director General of CSIR. The prize is distributed by the Prime Minister of India.
The awardee is bound to give a lecture in the area of the award, generally outside
his/her city of work.

TS

http://www.thehindu.com/news/national/11-selected-for-shanti-swarup-bhatnagar-a
ward/article7692862.ece
Q Source: SET A: Q138: CSP 2009

SI

86 The Vesara style of temple architecture is a combination of mainly


A. Dravida and Nagara styles
B. Hoysala and Halebidu styles
C. Nagara and Hoysala Styles
D. Dravida and Halebidu Styles
User Answer :
Correct Answer : A
Answer Justification :

IN

Learning: The Vesara style contain elements of both Dravida and Nagara styles.
The Vesara style is also described in some texts as the 'Central Indian temple
architecture style' or 'Deccan architecture'.
However many historian agree that the vesara style originated in what is today
Karnataka.
The trend was started by the Chalukyas of Badami (500-753AD) who built temples
in a style that was essentially a mixture of the nagara and the dravida styles, further
refined by the Rashtrakutas of Manyakheta (750-983AD) in Ellora, Chalukyas of
Kalyani (983-1195 AD) in Lakkundi, Dambal, Gadag etc. and epitomized by the
Hoysala empire
Q Source: Page 69: Chapter 6: 11th NCERT: An Introduction to Indian Arts

(C) Insights Active Learning. | All rights reserved.

www.insightsias.com

61

TEST - 12

IA
S

87 Consider the following about the Treaty of Madras 1769.


1. It was signed between the Mysore and the British East India Company.
2. The treaty allowed for the annexation of the signing Kingdom by the British.
3. It brought an end to the First Carnatic War.
Select the correct answer using the codes below.
A. 1 only
B. 2 and 3 only
C. 1 and 3 only
D. 1, 2 and 3
User Answer :
Correct Answer : A
Answer Justification :

TS

Justification & Learning:The Treaty of Madras was a peace agreement which


brought an end to the First Anglo-Mysore War. Fighting had broken out in 1767 and
the forces of Hyder Ali had come close to capturing Madras at one point.
The Treaty contained a clause requiring the British to assist Hyder Ali if he was
attacked by his neighbours.

Hyder felt this agreement was broken when he didn't receive any help when Mysore
went to war with the Marathas in 1771.

Bad faith arising from the broken clause may have been a reason behind the
outbreak of the Second Anglo-Mysore War a decade later

SI

Q Source: CAPF 2011

IN

88 Peitra Dura is
A. A decorative art made using polished coloured stones
B. A royal weaving pattern mentioned in Akbarnama
C. A handmade decorative blade fitted with gemstones to showcase military
victories
D. A miniature painting of natural colours often found in Mughal Darbars
User Answer :
Correct Answer : A
Answer Justification :
Learning: It is an art that flourished in the late 16th and 17th centuries and involved
the fashioning of highly illusionistic pictures out of cut-to-shape pieces of coloured
stone.

(C) Insights Active Learning. | All rights reserved.

www.insightsias.com

62

TEST - 12

The resulting decorative mosaics were used primarily for tabletops and small wall
panels.

TS

IA
S

The term pietra dura signifies the requisite hardness and durability of the materials
used in this work

Q Source: Page 112: Chapter 8: 11th NCERT: An Introduction to Indian Arts

IN

SI

89 "2000K" written over a light bulb denotes which of the following?


A. Longevity of the bulb
B. Electricity consumption of the bulb
C. Recycling ability of the bulb
D. Colour Temperature of the Light
User Answer :
Correct Answer : D
Answer Justification :
Learning: Every light source has a distinct character, from the warm, dim glow of a
candle to the blue, bright beam of a street light.
Brightness, measured in lumens, is one part of that character; the other part is color
temperature.
Measured in degrees Kelvin, color temperature is not the ambient hot/cold
temperature of our surroundings. In fact, the Kelvin scale goes backwards: The
higher the color temperature, the cooler light gets, and the lower the color

(C) Insights Active Learning. | All rights reserved.

www.insightsias.com

63

TEST - 12

temperature, the warmer light gets.


2000K is a warm colour temperature light.
Q Source: CSP 2006

TS

IA
S

90 Consider the following statements about the Cabinet Secretary.


1. She is the ex-officio Chairman of the Civil Services Board.
2. She chairs the Cabinet in the absence of the Prime Minister.
Which of the above is/are correct?
A. 1 only
B. 2 only
C. Both 1 and 2
D. None
User Answer :
Correct Answer : A
Answer Justification :

Justification & Learning: Cabinet Secretary is also the head of the civil services.
The Secretaries felt it necessary to keep the Cabinet Secretary informed of
developments from time to time. So, 1 is correct.

SI

Meetings of Cabinet are chaired by the Prime Minister (or the next most senior
Minister present, if the Prime Minister is absent). Cabinet committees are chaired by
the designated chair (or, in the chair's absence, the most senior committee member
present). So, 2 is incorrect.

IN

Committees are constituted for discussing specific matters and proposals emanating
from various Secretaries to the Government and meetings are held under the
chairmanship of the Cabinet Secretary.
Among the inter-Ministerial matters, the coordination is required for:
Removing difficulties.
Removing differences.
Overcoming delays.
Coordination in administrative action.
Coordination of policies.

These are achieved with the help of the Cabinet Secretary.

(C) Insights Active Learning. | All rights reserved.

www.insightsias.com

64

TEST - 12

Q Source: SET A: Improvisation: Q94: CSP 2009

IA
S

91 Operation Enduring Freedom (OEF) is


A. The Global War on Terrorism by the Government of the United States of
America
B. 2003 Invasion of Iraq by the USA
C. Bombings of Afghanistan and Sudan bases by American forces
D. The Gulf War operations leading to the buildup of troops in Saudi Arabia
User Answer :
Correct Answer : A
Answer Justification :

TS

Learning: The U.S. government used the term "Operation Enduring Freedom Afghanistan" to officially describe the War in Afghanistan, from the period between
October 2001 and December 2014.
Continued operations in Afghanistan by the United States' military forces, both noncombat and combat, now occur under the name Operation Freedom's Sentinel.

Q Source: Page 50: Chapter 3: 12th NCERT: Contemporary World Politics

IN

SI

92 Consider the following statements about the European Union.


1. It is a political Union.
2. It is a sub-agency of NATO.
3. It has a common constitution.
4. There is a common flag and anthem.
Select the correct answer using the codes below.
A. 1 and 4 only
B. 2 and 3 only
C. 1, 3 and 4 only
D. 4 only
User Answer :
Correct Answer : D
Answer Justification :
Justification: Statement 1: It is an economic Union, not a political Union, as all
member states are autonomous units.
Statement 2: Some EU members are also NATO members, but EU is not its subagency.

(C) Insights Active Learning. | All rights reserved.

www.insightsias.com

65

TEST - 12

Statement 3: The Treaty establishing a Constitution for Europe (TCE), (commonly


referred to as the European Constitution or as the Constitutional Treaty), was an
unratified international treaty intended to create a consolidated constitution for the
European Union (EU). It would have replaced the existing European Union treaties
with a single text. But it was rejected by the French and Dutch voters.

IA
S

Statement 4: It has its own flag, anthem, founding date, and currency. It also has
some form of a common foreign and security policy in its dealings with other
nations.
Q Source: Page 52: Chapter 3: 12th NCERT: Contemporary World Politics

SI

TS

93 Which of the following is/are landlocked countries with a monarchy?


1. Afghanistan
2. Bolivia
3. Kyrgyzstan
4. Zimbabwe
Select the correct answer using the codes below.
A. 2 and 4 only
B. 1 and 3 only
C. 2 only
D. None of the above
User Answer :
Correct Answer : D
Answer Justification :
Justification: All countries are landlocked countries.

IN

Zimbabwe is a Dominant-party presidential republic.


Kyrgyzstan is a unitary parliamentary republic.
Bolivia is a unitary presidential constitutional republic. Afghanistan is a unitary
presidential Islamic republic.
Q Source: Chapter 5: 12th NCERT: Contemporary World Politics

94 Which of the following is/are related to Buddhism?


1. Kanheri Caves
2. Karla Caves

(C) Insights Active Learning. | All rights reserved.

www.insightsias.com

66

TEST - 12

IA
S

3. Bhaja Caves
4. Junnar Caves
5. Bedse Caves
Select the correct answer using the codes below.
A. 1, 2 and 3 only
B. 1 and 2 only
C. 3, 4 and 5 only
D. 1, 2, 3, 4 and 5
User Answer :
Correct Answer : D
Answer Justification :

TS

Justification: The stupas in the fourth and fifth centuries CE have Buddha images
attached. Junnar has the largest cave excavations more than two hundred caves
around the hills of the townwhereas Kanheri in Mumbai has a hundred and eight
excavated caves.
The most important sites are Ajanta, Pitalkhora, Ellora, Nashik, Bhaja, Junnar,
Karla, Kanheri. Ajanta, Ellora, and Kanheri continue to flourish.

Earlier it was presumed that because of the absence of the Buddha image, the caves
were considered belonging to the orthodox faith of Buddhism, i.e., the Thervadins,
but with the discovery of the Konkan Maurya inscription mentioning the Saka era
322, i.e., 400 CE, it is now satisfactorily proved that the cave activity in western
Deccan was an ongoing process.

SI

Q Source: Page 37: Chapter 4: 11th NCERT: An Introduction to Indian Arts

IN

95 To implement Basel III norms, Indian banks will have to


1. Relinquish government control completely
2. Float all their capital in money markets
3. Allow a minimum foreign holding in every bank
Select the correct answer using the codes below.
A. 1 and 2 only
B. 2 and 3 only
C. 3 only
D. None of the above
User Answer :
Correct Answer : D
Answer Justification :

(C) Insights Active Learning. | All rights reserved.

www.insightsias.com

67

TEST - 12

Justification: The Basel III framework, whose main thrust has been enhancing the
banking sector's safety and stability, emphasises the need to improve the quality and
quantity of capital components, leverage (lending debt) ratio, liquidity standards,
and enhanced disclosures (transparency).

IA
S

It does not recommend a clean sweep privatization of banks or floating all reserves
in markets or necessary FDI stockholding in the bank. So, all statements are
incorrect.
Q Source: CSP 2007

TS

96 Consider the following statements about the Non-alignment Movement (NAM) Summit.
1. It is customary for the United Nations (UN) Secretary General to chair the summit.
2. Serving or Former Heads of the State cannot be appointed as Secretary General of
the summit.
Which of the above is/are correct?
A. 1 only
B. 2 only
C. Both 1 and 2
D. None
User Answer :
Correct Answer : D
Answer Justification :

SI

Justification: The countries of the Non-Aligned Movement represent nearly twothirds of the United Nation's members. The UN Sec. Gen. can attend NAM summits,
but is not mandatory for them to attend it under any convention or international law.
Nor is it customary for them to chair it. So, 1 is wrong.

IN

Many heads of states have been appointed as NAM Sec. Gen, e.g. Neelam Sanjeev
Reddy, Zail Singh etc. So, 2 is also wrong.
Learning: Since the end of the Cold War and the formal end of colonialism, the
Non-Aligned Movement has been forced to redefine itself and reinvent its purpose
in the current world system.
In recent years the organization has criticized certain aspects of US foreign policy.
The 2003 invasion of Iraq and the War on Terrorism, its attempts to stifle Iran and
North Korea's nuclear plans, and its other actions have been denounced by some
members of the Non-Aligned Movement as attempts to run roughshod over the
sovereignty of smaller nations.

(C) Insights Active Learning. | All rights reserved.

www.insightsias.com

68

TEST - 12

Q Source: SET A: Improvisation: Q1: CSP 2009

TS

IA
S

97 Consider the following statements about the Cabinet Secretariat.


1. It is under the direct charge of the Union Minister of Personnel.
2. It is the custodian of the papers of the Cabinet meetings.
3. It watches the implementation of the decisions taken by the Cabinet.
4. Management of major crisis situations in the country is one of its functions.
Select the correct answer using the codes below.
A. 2, 3 and 4 only
B. 3 and 4 only
C. 2 and 3 only
D. 1, 2, 3 and 4
User Answer :
Correct Answer : A
Answer Justification :

Justification & Learning: The Cabinet Secretariat is responsible for the


administration of the Government of India (Transaction of Business) Rules, 1961
and the Government of India (Allocation of Business) Rules 1961, facilitating
smooth transaction of business in Ministries/ Departments of the Government by
ensuring adherence to these rules. It is under the Direct Charge of the Prime
Minister. So, 1 is wrong.

IN

SI

The Secretariat assists in decision-making in Government by ensuring InterMinisterial coordination, ironing out differences amongst Ministries/
Departments and evolving consensus through the instrumentality of the
standing/ adhoc Committees of Secretaries.
The Cabinet Secretariat ensures that the President, the Vice President and
Ministers are kept informed of the major activities of all
Ministries/Departments by means of monthly summary of their activities.
Management of major crisis situations in the country and coordinating
activities of various Ministries in such a situation is also one of the functions
of the Cabinet Secretariat.
The administrative head of the Cabinet Secretariat is the Cabinet Secretary.

Q Source: SET A: Improvisation: Q94: CSP 2009


98 The source(s) of exposure to Bisphenol A (BPA) for humans is
1. Leaching into food from food storage containers
2. Through air and skin absorption

(C) Insights Active Learning. | All rights reserved.

www.insightsias.com

69

Which of the above is/are correct?


A. 1 only
B. 2 only
C. Both 1 and 2
D. None
User Answer :
Correct Answer : C
Answer Justification :

IA
S

TEST - 12

Justification: What is it: Bisphenol A (BPA) is a chemical produced in large


quantities for use primarily in the production of polycarbonate plastics and epoxy
resins.

TS

Statement 1: Bisphenol A can leach into food from the protective internal epoxy
resin coatings of canned foods and from consumer products such as polycarbonate
tableware, food storage containers, water bottles, and baby bottles. The degree to
which BPA leaches from polycarbonate bottles into liquid may depend more on the
temperature of the liquid or bottle, than the age of the container.

Statement 2: Besides diet, exposure can also occur through air and through skin
absorption. Free BPA is found in high concentration in thermal paper and carbonless
copy paper, which would be expected to be more available for exposure than BPA
bound into resin or plastic. Popular uses of thermal paper include receipts, event and
cinema tickets, labels, and airline tickets

SI

Q Source: Q on Bisphenol A: CSP 2008

IN

99 Which of the following is/are NOT banned in India concerning Internet?


1. Charging discriminatory tariffs for data services on the basis of content accessed by
a consumer.
2. Providing free content on websites to selected consumers.
Select the correct answer using the codes below.
A. 1 only
B. 2 only
C. Both 1 and 2
D. None
User Answer :
Correct Answer : B
Answer Justification :
Justification: Statement 1: The Telecom Regulatory Authority of India (TRAI) has

(C) Insights Active Learning. | All rights reserved.

www.insightsias.com

70

TEST - 12

barred telecom service providers (TSPs) from offering or charging such


discriminatory tariffs, as it went against Net Neutrality.
Statement 2: It is a standard practice. For e.g. EPW magazine provides free access
and content to academic scholars and students from selected universities, whereas
for the same content it charges others.

IA
S

The concept of net neutrality has been covered in a test earlier.

Q Source:
http://indianexpress.com/article/technology/tech-news-technology/trai-differential-p
ricing-order-explained-its-beyond-facebook-and-free-basics/

TS

100 The term North-South divide in the context of International affairs is related to
A. A socio-economic and political divide between the Developed and
Developing World
B. Differentiating between the Aggressor states and invaded states
C. Former colonial occupations of Western powers in the Southern Hemisphere
D. Diplomatic groupings formed within the Asian nations
User Answer :
Correct Answer : A
Answer Justification :

IN

SI

Learning: The North mostly covers the West and the First World, along with much
of the Second World, while the South largely corresponds with the Third World.
While the North may be defined as the richer, more developed region and the
South as the poorer, less developed region, many more factors differentiate
between the two global areas. 95% of the North has enough food and shelter.
Similarly, 95% of the North has a functioning educational system. In the
South, on the other hand, only 5% of the population has enough food and
shelter.
It "lacks appropriate technology, it has no political stability, the economies
are disarticulated, and their foreign exchange earnings depend on primary
product exports".
As nations become economically developed, they may become part of the
"North", regardless of geographical location, while any other nations which
do not qualify for "developed" status are in effect deemed to be part of the
"South".

Q Source: Chapter 8: 12th NCERT: Contemporary World Politics

(C) Insights Active Learning. | All rights reserved.

www.insightsias.com

71

You might also like